Sie sind auf Seite 1von 167

Question: 1 of 148 Time taken: 01:35

This 35-year-old woman developed this rash associated with facial swelling three weeks after she was started on oral carbamazepine for new-onset epilepsy. he was also fo!nd to be febrile and had raised liver enzymes. "hat treatment will she re#!ire$ %&lease select 1 option' (igh-dose oral corticosteroids )orrect)orrect *ntraveno!s imm!noglob!lins +ral antibiotics +ral anti-f!ngals Topical corticosteroids This patient has dr!g reaction with eosinophilia and systemic symptoms or ,-. syndrome. *t is a severe idiosyncratic dr!g reaction/ characterised by a generalised erythemato!s rash often associated with facial oedema/ involvement of internal organs %liver dysf!nction'/ haematologic abnormalities %eosinophilia' and systemic illness %fever'. *t is treated with high dose oral corticosteroids for several months. *ntraveno!s imm!noglob!lins %*0*1' are not indicated in the treatment of ,-. syndrome. ome centres !se *0*1 for treatment of to2ic epidermal necrolysis. 1

+ral antibiotics are not !sed to treat ,-.

syndrome. syndrome. syndrome.

+ral anti-f!ngals are not indicated in the treatment of ,-. Topical corticosteroids are not !sef!l in the treatment of ,-. ------------------Question: 2 of 148 Time taken: 03:34

This 35-year-old woman developed this rash associated with facial swelling three weeks after she was started on oral carbamazepine for new-onset epilepsy. he was also fo!nd to be febrile and had raised liver enzymes. "hat is her diagnosis$ %&lease select 1 option' 5ngioedema *ncorrect*ncorrect answer selected ,r!g reaction with eosinophilia and systemic symptoms %,-. ' This is the correct anwserThis is the correct answer 6i2ed dr!g er!ption To2ic epidermal necrolysis 0iral e2anthem This patient has ,-. syndrome/ also known as dr!g hypersensitivity syndrome. *t is a severe idiosyncratic dr!g reaction/ characterised by a generalised erythemato!s rash often associated with facial oedema/ involvement of internal organs %liver dysf!nction'/ haematologic abnormalities %eosinophilia' and systemic illness %fever'. 7

*t is treated with high dose oral corticosteroids for several months. )ommon dr!gs implicated incl!de the anti-epileptics/ s!lph!r dr!gs and the penicillins. 5ngioedema secondary to dr!g ingestion can also present with facial swelling b!t it occ!rs after a short latency period after starting the dr!g. &atients may also have associated !rticarial lesions. 6i2ed dr!g er!ption presents with one or several erythemato!s-to-d!sky patches or pla#!es with central blister. *t is not associated with systemic symptoms. To2ic epidermal necrolysis is another severe idiosyncratic dr!g reaction that presents with e2tensive epidermal necrosis and m!cosal involvement. 0iral e2anthems may also present with a similar rash and systemic symptoms b!t a dr!g er!ption is more likely in this case with the history of ingestion of oral anti-epileptics. ----------------------------------------------Question: 3 of 148 Time taken: 08:33

This 95-year-old man presents with these lesions on his face for the past three years. The lesions are worse after s!n e2pos!re. 5 skin biopsy will reveal which of the following histological patterns$ %&lease select 1 option' *nterface dermatitis )orrect)orrect 3

*ntraepidermal b!lla &annic!litis pongiotic dermatitis !bepidermal b!lla This patient has discoid l!p!s erythematos!s. 5 skin biopsy will reveal interface dermatitis/ characterised by basal vac!olar change/ necrotic keratinocytes and pigment incontinence. This reaction represents damage to the epidermis from the inflammation. 5n intraepidermal b!lla is seen in intraepidermal blistering disorders/ for e2ample/ pemphig!s. &annic!litis is inflammation of the s!bc!taneo!s fat and is seen classically in erythema nodos!m. pongiotic dermatitis is the histological pattern seen in eczemas and other pap!los#!amo!s disorders/ for e2ample/ pityriasis rosea. 5 s!bepidermal b!lla is seen in s!bepidermal blistering disorders/ for e2ample/ b!llo!s pemphigoid. --------------------------------------------------Question: 4 of 148 Time taken: 10:93

This child has this facial abnormality present since birth. "hat other medical problems might she s!ffer from$ 9

%&lease select 1 option' 5ortic coarctation *ncorrect*ncorrect answer selected .pilepsy This is the correct anwserThis is the correct answer :iver haemangiomas &osterior fossa abnormality -enal artery stenosis This patient has t!rge-"eber syndrome which presents with a facial port wine stain of the 01 ;<- 07 or 03 segments. +ther abnormalities incl!de epilepsy/ developmental delay and gla!coma. 5ortic coarctation is seen in &(5). syndrome which presents with segmental facial haemangioma/ not a port wine stain. :iver haemangiomas are seen in some patients with m!ltiple c!taneo!s haemangiomas. &osterior fossa abnormality/ for e2ample/ ,andy-"alker malformation/ is seen in patients with &(5). syndrome. -enal artery stenosis is seen in ne!rofibromatosis. -----------------------------------Question: 5 of 148 Time taken: 17:13

This 79-year-old woman presents with this rash for the past two weeks. he also complains of bilateral knee and ankle pains for the same d!ration of time. 5

"hat finding will be shown on a skin biopsy$ %&lease select 1 option' 6ibrinoid necrosis and ne!trophils within the walls of dermal capillaries )orrect)orrect *ntraepidermal clefting with eosinophils =e!trophils within walls of medi!m sized arteries in the s!bc!tis eptal pannic!litis !bepidermal b!lla with eosinophils The photograph reveals palpable p!rp!ra in a case of (enoch- chonlein p!rp!ra %note the symtoms of arthralgia' on both legs. :e!cocytoclastic vasc!litis or (ypersensitivity angitis/ histopathology reveals 6ibrinoid degeneration/ a ne!trophilic infiltrate aro!nd small blood vessels along with karyorhe2is or n!clear d!st arising o!t of disintegration of ne!trophils. The *ntraepidermal clefting with eosinophils histological pattern is consistent with intraepidermal blistering conditions/ for e2ample/ pemphig!s. =e!trophils within walls of medi!m sized arteries in the s!bc!tis is consistent with vasc!litis affecting medi!m-sized vessels/ most commonly polyarteritis nodosa. eptal pannic!litis is consistent with erythema nodos!m. !bepidermal b!lla with eosinophils is consistent with s!bepidermal blistering disorders/ for e2ample/ b!llo!s pemphigoid. ----------------------------------->!estion: ? of 193 Time taken: 13:74 This 3-year-old boy presents with a bizarre pattern of non-scarring hair loss which started abo!t one year ago. (e has no other systemic complaints.

+n e2amination/ yo! notice short hairs of different lengths within the areas of alopecia. "hat is yo!r most likely diagnosis$ %&lease select 1 option' 5ndrogenetic alopecia *ncorrect*ncorrect answer selected 5lopecia areata econdary syphilis Telogen effl!vi!m Trichotillomania This is the correct anwserThis is the correct answer Trichotillomania is more commonly seen in children and adolescents compared to ad!lts. *t is regarded as a primary psychiatric disorder and res!lts from repetitive hair manip!lation by the patient@s own hands. *t res!lts in a patchy non-scarring alopecia that often has a bizarre distrib!tion. mall/ broken hairs of varying lengths may be seen within the patches of alopecia. 5ndrogenetic alopecia does not occ!r in the pre-p!bertal age gro!p. 5lopecia areata is a differential diagnosis to consider in patients presenting with trichotillomania. (owever/ it !s!ally does not lead to a @bizarre@ distrib!tion and the patches of alopecia are !s!ally @clean@ witho!t short/ broken/ hairs of varying lengths. econdary syphilis may lead to a non-scarring alopecia with a @moth-eaten@ appearance. (owever/ it occ!rs in se2!ally active persons. Telogen effl!vi!m occ!rs one to three months after a maAor stress to the body/ for e2ample/ illness/ s!rgery/ child birth. *t leads to a diff!se form of non-scarring alopecia. ------------------------------------------------>!estion: 4 of 193

Time taken: 13:3? 5 1?-year-old boy has severe inflammatory acne that has not responded to treatment with oral antibiotics. (is dermatologist has decided to start him on oral isotretinoin. "hich of the following pairs of laboratory tests are re#!ired before and d!ring treatment with oral isotretinoin$ %&lease select 1 option' :iver f!nction tests and fasting lipid levels )orrect)orrect :iver f!nction tests and platelet levels &latelet levels and ser!m electrolytes er!m electrolytes and fasting lipid levels er!m electrolytes and liver f!nction tests :iver f!nction tests and fasting lipid levels are the correct tests. The other pairs of laboratory tests are not re#!ired d!ring treatment with oral isotretinoin. ----------------------------------->!estion: 3 of 193 Time taken: 19:14 5 17-year-old girl has severe atopic dermatitis which has not been well controlled with topical treatments/ despite good compliance. (er dermatologist has decided to start her on treatment with oral ciclosporin. "hich of the following parameters needs to be reg!larly monitored while she is on treatment with ciclosporin$ %&lease select 1 option' Blood gl!cose levels *ncorrect*ncorrect answer selected Blood press!re This is the correct anwserThis is the correct answer 3

Bone age (eight &!bertal staging Blood gl!cose levels/ bone age/ height and p!bertal staging are not affected by ciclosporin therapy. ---------------------------------------------->!estion: 8 of 193 Time taken: 1?:90 5 75-year-old woman has A!st been diagnosed with chronic c!taneo!s l!p!s erythematos!s. he has no other clinical or laboratory evidence to s!ggest systemic involvement. 5part from advice on s!n avoidance/ her dermatologist has decided to start her on oral hydro2ychloro#!ine. "hile on treatment with hydro2ychloro#!ine/ she re#!ires pre-treatment eval!ation and reg!lar monitoring by which of the following specialists$ %&lease select 1 option' )ardiologist *ncorrect*ncorrect answer selected 1astroenterologist (aematologist =e!rologist +phthalmologist This is the correct anwserThis is the correct answer .ye to2icity is one of the most serio!s side effects of treatment with antimalarials and re#!ires reg!lar screening. The cornea and mac!la may be affected by antimalarial medications. -eg!lar monitoring by specialists other than an ophthalmologist is not warranted for patients on treatment with antimalarials/ altho!gh their services may be re#!ired in the event of the !ncommon occ!rrence of systemic involvement of l!p!s in this patient. 8

------------------------------->!estion: 10 of 193 Time taken: 14:03 5 35-year-old man has A!st been diagnosed with dermatitis herpetiformis. Besides starting on a gl!ten-free diet/ his dermatologist has decided to start him on oral dapsone. "hat laboratory test needs to be within the normal range before commencing therapy$ %&lease select 1 option' 6asting gl!cose *ncorrect*ncorrect answer selected 6asting lipids 1l!cose-?-phosphate dehydrogenase %1?&,' levels This is the correct anwserThis is the correct answer (aemoglobin-51) levels %(b51)' Thiop!rine methyltransferase %T&CT' levels 1?&, deficiency is an absol!te contraindication to treatment with dapsone as it can lead to severe haemolytic anaemia. This is probably d!e to the =hydro2y metabolites of dapsone/ which are direct haemolytic agents. "hen allowed to b!ild !p to large eno!gh amonths/ they ind!ce premat!re se#!estration of the red cell in the spleen. 5bnormal (b51)/ fasting lipids and fasting gl!cose levels are not considered contraindications to treatment with dapsone. T&CT levels are !sed to decide on starting doses of azathioprine. ---------------------------------------Question: 11 of 148 Time taken: 14:7? 10

This ?-month-old infant/ who was born premat!rely/ presents with this lesion on his arm which was first noticed d!ring the second week of life. *t then progressively enlarged and deepened in colo!r over the ne2t few months. "hat is yo!r most likely diagnosis$ %&lease select 1 option' 5ngiosarcoma *ncorrect*ncorrect answer selected (aemangioma This is the correct anwserThis is the correct answer Dasabach-Cerritt syndrome :ymphatic malformation 0eno!s malformation *nfantile haemangiomas are common vasc!lar t!mo!rs that present in early infancy. They contin!e to enlarge and deepen in colo!r for the first few months of life before slowly invol!ting after si2 to 17 months. They are more common in premat!re infants. 5ngiosarcomas/ a high grade vasc!lar malignancy/ are very !ncommon in infancy. Dasabach-Cerritt syndrome is seen in patients with kaposiform haemangioendothelioma/ a rare vasc!lar t!mo!r seen in infants that present as a large/ firm/ reddish-bl!e t!mo!r. &atients !s!ally have thrombocytopenia. :ymphatic malformations are !s!ally well formed at birth and do not have a proliferative stage like haemangiomas. They are !s!ally skin colo!red or bl!ish in appearance.

11

0eno!s malformations are !s!ally well formed at birth and do not have a proliferative stage like haemangiomas. )linically/ they are !s!ally skin colo!red or have a bl!ish h!e. -----------------------------------------------------------Question: 12 of 148 Time taken: 30:78

This 13-year-old man presents with this rec!rrent rash on both feet for the past two years. *t worsened when football training started. (e also has a history of allergic rhinitis. "hat is yo!r most likely diagnosis$ %&lease select 1 option' B!llo!s pemphigoid *ncorrect*ncorrect answer selected B!llo!s tinea pedis .rythema m!ltiforme &ompholy2 < dyshidrotic eczema This is the correct anwserThis is the correct answer Eoster &ompholy2 or dyshidrotic eczema presents bilaterally on the hands and feet as itchy/ erythemato!s blisters that resemble @sago seeds@. *t is commonly seen in yo!ng patients with a history of atopy and is more common in individ!als who wear closed shoes or boots for long periods. B!llo!s pemphigoid is very !ncommon in this age gro!p. B!llo!s tinea pedis !s!ally presents asymmetrically on one foot.

17

.rythema m!ltiforme may present with acral blisters b!t are most commonly targetoid lesions. There is associated m!cosal involvement. Eoster presents as blisters in a dermatomal distrib!tion/ !s!ally asymmetrically. --------------------------------------------------Question: 13 of 148 Time taken: 98:34

5n 13-year-old girl presents with this rash on both her !pper arms and thighs for the past few years. *t is generally asymptomatic. "hich of the following may be an associated skin condition$ %&lease select 1 option' 5topic dermatitis )orrect)orrect :ichen plan!s =od!locystic acne &ilonidal sin!s &soriasis v!lgaris This patient has keratosis pilaris/ characterised by tiny/ hyperkeratotic/ follic!lar pap!les/ most commonly affecting the !pper arms and thighs. *t is commonly seen in patients with atopic dermatitis. There is no proven association with the other conditions listed here. ----------------------------------------------Question: 14 of 148 13

Time taken: 50:30

5n elderly patient presented with an ac!te onset painf!l rash %above' on the face. "hat is the most likely diagnosis$ %&lease select 1 option' 5llergic contact dermatitis *ncorrect*ncorrect answer selected 5topic dermatitis .rythema m!ltiforme (erpes zoster This is the correct anwserThis is the correct answer *rritant contact dermatitis (erpes zoster presents with gro!ped vesicob!llo!s lesions in a dermatomal distrib!tion. 5llergic contact dermatitis may present with an ac!te eczemato!s reaction at the site of contact with the allergen after a period of sensitisation. 5topic dermatitis may present in the ac!te phase with an ac!te eczemato!s reaction on the face in infants. *n ad!lts the fle2!res are involved.

19

.rythema m!ltiforme classically presents with target lesions with a central b!llo!s lesion s!rro!nded by an erythemato!s halo following the intake of an offending dr!g. *rritant contact dermatitis presents with an ac!te eczemato!s reaction with erythema/ vesic!lation/ cr!sting and itching or a b!rning sensation at the site of the contact of the allergen. 5 sensitisation dose is not re#!ired as in allergic contact dermatitis. -------------------------------------------Question: 15 of 148 Time taken: 51:13

5 middle aged female patient presented with a non-itchy scaly rash on the scalp/ elbows and knees. "hat is the most likely diagnosis$ %&lease select 1 option' 5llergic contact dermatitis *ncorrect*ncorrect answer selected 5topic dermatitis *rritant contact dermatitis &soriasis This is the correct anwserThis is the correct answer Tinea capitis

15

&soriasis v!lgaris presents as scaly pla#!es on the e2tensors of limbs/ scalp/ palms and soles. +n the scalp it presents as non-itchy scaly pla#!es witho!t loss of hair. 5llergic contact dermatitis may present with an ac!te eczemato!s reaction at the site of contact with the allergen after a period of sensitisation. 5topic dermatitis may present in the ac!te phase with an ac!te eczemato!s reaction on the face in infants. *n ad!lts the fle2!res are involved. *rritant contact dermatitis presents as an ac!te eczemato!s reaction at the site of contact with the irritant. Tinea capitis presents as itchy scaly pla#!es on the scalp along with loss of hair/ presenting as @black-dot@ alopecia. -------------------------------------Question: 16 of 148 Time taken: 57:05

5 yo!ng male presented with an itchy rash %above' on the b!ttocks and groins. "hat is the most likely diagnosis$ %&lease select 1 option' --------------------------------------------------------1?

5llergic contact dermatitis *ncorrect*ncorrect answer selected 5topic dermatitis ,ermatophytosis This is the correct anwserThis is the correct answer *rritant contact dermatitis cabies ,ermatophytosis is ca!sed by a f!ngal infetion of the skin. *t presents with itchy ann!lar pla#!es with a clear centre and a spreading peripheral margin. *t is termed tinea corporis when it affects the tr!nk or e2tremities and tinea cr!ris when it affects the natal cleft or groins. 5llergic contact dermatitis may present with an ac!te eczemato!s reaction at the site of contact with the allergen after a period of sensitisation. 5topic dermatitis may present in the ac!te phase with an ac!te eczemato!s reaction on the face in infants. *n ad!lts the fle2!res are involved. *rritant contact dermatitis presents with an ac!te eczemato!s reaction with erythema/ vesic!lation/ cr!sting and itching or a b!rning sensation at the site of the contact of the irritant. 5 sensitisation dose is not re#!ired as in allergic contact dermatitis. cabies presents as itchy pap!lar er!ptions and b!rrows in the fle2!res incl!ding the web spaces. ----------------------------------------Question: 17 of 148 Time taken: 53:17

14

5 yo!ng female patient presented with an ac!te onset itchy rash on the face following the !se of a new face cream for the first time. "hat is the most likely diagnosis$ %&lease select 1 option' 5llergic contact dermatitis *ncorrect*ncorrect answer selected 5topic dermatitis .rythema m!ltiforme (erpes zoster *rritant contact dermatitis This is the correct anwserThis is the correct answer *rritant contact dermatitis presents with an ac!te eczemato!s reaction with erythema/ vesic!lation/ cr!sting and itching or a b!rning sensation at the site of the contact of the irritant. 5 sensitisation dose is not re#!ired as in allergic contact dermatitis. *rritant contact dermatitis occ!rs !pon e2pos!re to a s!bstance either in s!fficient #!antity or when the skin is !nd!ly sensitive to the s!bstance in which case it shall react to it in small amo!nts. (ence no prior sensitization is needed in irritant contact dermatitis. (owever the skin may react to every s!bse#!ent e2pos!re too/ depending on it@s sensitivity. 5llergic contact dermatitis is a delayed type hypersensitivity developing against a specific allergen to which the skin has been sensitized to. 13

5llergic contact dermatitis may present with an ac!te eczemato!s reaction at the site of contact with the allergen after a period of sensitisation. 5topic dermatitis may present in the ac!te phase with an ac!te eczemato!s reaction on the face in infants. *n ad!lts the fle2!res are involved. .rythema m!ltiforme classically presents with target lesions with a central b!llo!s lesion s!rro!nded by an erythemato!s halo following the intake of an offending dr!g. (erpes zoster presents with vesicob!llo!s lesions in a dermatomal distrib!tion. ------------------------------------Question: 18 of 148 Time taken: 53:94

5 yo!ng male presented with m!ltiple non-tender !mbilicated pap!les in the s!prap!bic region and the scrot!m. "hat is the most likely diagnosis$ %&lease select 1 option' 6ollic!litis *ncorrect*ncorrect answer selected :ichen plan!s 18

Coll!sc!m contagios!m This is the correct anwserThis is the correct answer &soriasis 0err!ca v!lgaris Coll!sc!m contagios!m presents as m!ltiple !mbilicated pap!les in children/ health care workers and spreads by direct contact and a!to inoc!lation. *n the imm!nocompromised it may attain a very large size. *n the se2!ally active protected e2pos!re to an infected partner spares the penile shaft in the male and may infect the s!rro!nding area. having the affected part spreads the infection by a!toinoc!lation. 6ollic!litis is a painf!l p!st!lar er!ption with follic!lar involvement. :ichen plan!s presents as itchy violaceo!s polygonal pap!lar lesions commonly over the ankles/ wrists and low back. &soriasis presents as pap!los#!amo!s non-itchy pla#!es which e2hibit micaceo!s scaling and !s!ally occ!r over the e2tensors of the e2tremities. 0err!ca v!lgaris or common warts present as solitary or m!ltiple painless skin lesions ca!sed by h!man papillomavir!s. ---------------------------------------------------------------------Question: 19 of 148 Time taken: 59:09

70

5 yo!ng male presented with m!ltiple nod!locystic lesions and comedones over the face. "hat is the most likely diagnosis$ %&lease select 1 option' 5cne )orrect)orrect 6ollic!litis (idradenitis s!pp!rativa &yoderma faciale ycosis barbae =od!locystic acne presents as m!ltiple nod!les/ cysts and interconnecting lesions over the face. 6ollic!litis is a painf!l p!st!lar er!ption with follic!lar involvement. (idradenitis s!pp!rativa presents as tender erythemato!s pap!les/ abscesses and dermal contract!res in the apocrine gland bearing areas of the skin s!ch as the a2illary regions. &yoderma faciale a rare complication of rosacea presents with m!ltiple nod!les/ abscesses and sin!s tracts along with systemic signs. )omedones are absent. 71

ycosis barbae presents as painf!l p!st!lar er!ptions in the glabro!s areas of the skin. ------------------------------------Question: 20 of 148 Time taken: 59:30

5 yo!ng female presented with m!ltiple non-tender/ non-scaly/ verr!co!s lesions on the leg and foot. "hat is the most likely diagnosis$ %&lease select 1 option' 6ollic!litis *ncorrect*ncorrect answer selected :ichen plan!s Coll!sc!m contagios!m &soriasis 0err!ca v!lgaris This is the correct anwserThis is the correct answer 0err!ca v!lgaris or common warts present as solitary or m!ltiple painless skin lesions ca!sed by h!man papillomavir!s. 6ollic!litis is a painf!l p!st!lar er!ption with follic!lar involvement. :ichen plan!s presents as itchy violaceo!s polygonal pap!lar lesions commonly over the ankles/ wrists and low back. Coll!sc!m contagios!m presents !s!ally as m!ltiple !mbilicated pap!les. 77

&soriasis presents as pap!los#!amo!s non-itchy pla#!es which e2hibit micaceo!s scaling and !s!ally occ!r over the e2tensors of the e2tremities. ------------------------------------------------------Question: 21 of 148 Time taken: 55:09

5 yo!ng farmer presented with web space infection as shown. "hat is the most likely ca!sative organism$ %&lease select 1 option' )andida *ncorrect*ncorrect answer selected ,ermatophytes &se!domonas This is the correct anwserThis is the correct answer taphylococc!s treptococc!s &se!domonas intertrigo typically presents with a bl!ish to greenish pigmentation in the affected areas. *n the typical @immersion foot@ the affected area is sodden and macerated with inhibition of the 1ram positive bacteria and dermatophytes. econdary invasion after pse!odomal infection is possible with )andida. )andidal web space infection presents as soft cottony growth in the macerated web space.

73

,ermatophyte infection of the web space res!lts in an itchy scaly lesion !s!ally with affliction of the forefoot or the dors!m with an ann!lar scaly pla#!e of tinea pedis. taphylococc!s and treptococc!s infections present with painf!l blistering maceration of the affected web space. ----------------------------------Question: 22 of 148 Time taken: 5?:19

5 patient presents with m!ltiple vesic!lar lesions s!rro!nded by erythema as shown following ingestion of s!lfametho2azole-trimethoprim. "hat is the most likely diagnosis$ %&lease select 1 option' .rythema m!ltiforme )orrect)orrect 6i2ed dr!g er!ption (erpes zoster *rritant contact dermatitis To2ic epidermal necrolysis .rythema m!ltiforme classically presents as discrete vesic!lar or b!llo!s lesions s!rro!nded by a pale area and a ring of erythema. (owever all three zones may not always be evident. The er!ption commonly follows the ingestion of the offending dr!g s!ch as those belonging to the s!lfa gro!p.

79

6i2ed dr!g er!ption may present with sharply circ!mscribed pigmented mac!les/ erythemato!s lesions or b!llo!s lesions with a classical history of rec!rrence at the same site following the ingestion of the offending dr!g. (erpes zoster presents as gro!ped vesic!lar er!ptions on an erythemato!s base in dermatomal distrib!tion. *rritant contact dermatitis presents with ac!te onset vesic!lar er!ptions at the site of contact of the irritant. To2ic epidermal necrolysis is an ac!te dermatological emergency which follows ingestion of offending dr!gs res!lting in tender erythema and widespread b!llae with s!bse#!ent skin necrosis and peeling. ----------------------------------Question: 23 of 148 Time taken: 5?:31

5 yo!ng male patient presented with sharply circ!mscribed hyperpigmented skin lesions over the back. "hat is the most likely diagnosis$ %&lease select 1 option' 5llergic contact dermatitis *ncorrect*ncorrect answer selected .rythema m!ltiforme 6i2ed dr!g er!ption This is the correct anwserThis is the correct answer 6ollic!litis :ichen plan!s

75

6i2ed dr!g er!ption may present with sharply circ!mscribed pigmented mac!les/ erythemato!s lesions or b!llo!s lesions with a classical history of rec!rrence at the same site following the ingestion of the offending dr!g. 5llergic contact dermatitis is a form of contact dermatitis which follows contact with the offending agent s!ch as chemicals/ nickel and plants like poison ivy. .rythema m!ltiforme classically presents as discrete vesic!lar or b!llo!s lesions s!rro!nded by a pale area and a ring of erythema. (owever all three zones may not always be evident. 6ollic!litis presents with painf!l pap!lop!st!lar follic!lar lesions. :ichen plan!s presents as violaceo!s polygonal pap!les !s!ally aro!nd the ankles and wrists. :ichen plan!s pigmentos!s may present in the dark skinned on the face and neck as dark to violaceo!s mac!les. ---------------------------------Question: 24 of 148 Time taken: 54:13 5 middle aged male presented with a lesion of si2 months@ d!ration on his neck %as shown' with a history of rec!rrent episodes of p!st!lation and scarring.

&lain radiograph of the chest revealed non-homogeno!s opacities in both l!ngs. "hat is the most likely diagnosis$ %&lease select 1 option' 7?

:!p!s v!lgaris )orrect)orrect Ciliary t!berc!losis crof!loderma #!amo!s cell carcinoma T!berc!lo!s chancre :!p!s v!lgaris is a chronic/ progressive form of c!taneo!s t!berc!losis characterised commonly by ann!lar pla#!es with areas of activity and scarring. :esions may either appear as a res!lt of endogeno!s spread of bacilli via haematogeno!s ro!te from an endogeno!s foc!s s!ch as p!lmonary t!berc!losis or at a site of e2ogeno!s direct inoc!lation. The lesion in the photograph represents one s!ch classic lesion. Ciliary t!berc!losis manifests as disseminated erythemato!s mac!les/ pap!les as well as p!rp!ric lesions/ d!e to haematogeno!s spread in a patient with t!berc!losis. crof!loderma presents initially as a s!bc!taneo!s nod!le which later breaks down to represent skin !lceration over an !nderlying area of t!berc!lo!s foc!s s!ch as a lymph node/ bone or Aoint. #!amo!s cell carcinoma typically arises from an !nhealthy skin !s!ally in areas of chronic photo-damage as an ind!rated scaly and fiss!red pla#!e. T!berc!lo!s chancre presents initially as an inflammatory pap!le at the site of inoc!lation which later breaks down into a non-healing/ shallow/ !ndermined !lcer with a gran!lomato!s base followed by painless regional lymphadenopathy. --------------------------------------Question: 25 of 148 Time taken: 54:5?

74

5 3-year-old child is bro!ght to yo! with a boggy swelling over the scalp with m!ltiple p!st!les s!rmo!nting the swelling along with alopecia. "hat is the most likely diagnosis$ %&lease select 1 option' 6av!s *ncorrect*ncorrect answer selected 6ollic!litis *mpetigo Derion This is the correct anwserThis is the correct answer eborrheic dermatitis Derion presents with an inflammatory swelling on the scalp or the glabro!s skin with n!mero!s p!st!les in response to zoophilic f!ngi res!lting in severe scarring alopecia in !ntreated cases. 6av!s/ a form of tinea capitis/ presents with lesions characterised by yellow c!p-shaped cr!sts termed sc!t!la/ which s!rro!nd the infected hair follicles. 6ollic!litis presents as painf!l follic!lar p!st!les. *mpetigo presents as vesic!lob!llo!s lesions or erosions covered with honey colo!red cr!sts. eborrheic dermatitis is an itchy scalp condition presenting with greasy scaling. ---------------------------------------73

Question: 26 of 148 Time taken: 53:99

5 yo!ng otherwise healthy adolescent with m!ltiple linear lacerations with the linear lesions shown presented in vario!s stages of healing with a vag!e history of possible antecedent ca!ses. "hat is the most likely primary diagnosis$ %&lease select 1 option' )ontact dermatitis *ncorrect*ncorrect answer selected ,ermatitis artefacta This is the correct anwserThis is the correct answer (ypertrophic scars Deloids triae distensae ,ermatitis artefacta typically presents in a healthy individ!al with !ne2plained skin lesions which may be bizarre/ sharply marginated geometric or linear tracks. Fnderlying psychiatric illness or a history of childhood neglect or ab!se may be present. )ontact dermatitis presents with itching/ vesic!lation or oozing in areas of contact with the offending allergen.

78

(ypertrophic scars follow inA!ries and typically do not e2tend beyond the margins of the wo!nd. 5s in keloids this case may present with m!ltiple hypertrophic scars/ however the primary diagnosis m!st be investigated. Deloids may develop as se#!elae to inA!ries where the overgrowth e2tends beyond the margins of the original inA!ry with claw-like e2tensions. *n this case s!ch a patient may present with m!ltiple keloids/ however the primary diagnosis m!st be investigated. .arly striae may present as pinkish linear lesions of thinned skin and mat!re striae as white irreg!larly shaped bands of depressed areas of skin with their long a2is parallel to lines of skin tension. ----------------------------------------Question: 27 of 148 Time taken: 58:94

5 yo!ng (*0 positive male presented with white discoloration of the nail plate. "hat is the most likely diagnosis$ %&lease select 1 option' )andidal onychomycosis )orrect)orrect )hronic paronychia *rritant contact dermatitis 30

&achyonychia congenita &soriatic nail dystrophy )andidal onychomycosis or white s!perficial onychomycosis altho!gh typically confined to the toenails may occ!r on the finger nails in the imm!nocompromised and manifests as small/ white/ speckled or powdery patches on the s!rface of the nail plate which gets ro!ghened and easily cr!mbles. )hronic paronychia presents with itchy painf!l swelling of the nail folds along with dystrophy of the nail plate. )hronic e2pos!re to irritants can lead to irreg!lar b!ckling and dystrophy of the nail plate. &achyonychia congenita presents with dystrophic/ thickened nails at or soon after birth along with palmoplantar keratoderma. &soriatic nail dystrophy presents with dystrophy of the nail plate/ s!b!ng!al hyperkeratosis/ oil spots/ onycholysis and splinter haemorrhages. -------------------------------------Question: 28 of 148 Time taken: 01:00:73

31

5 30-year-old female presented with mildly itchy violaceo!s pap!lar er!ptions on the dors!m of feet/ ankles/ wrist and low back. "hat is the most likely diagnosis$ %&lease select 1 option' :ichen plan!s )orrect)orrect &ap!lar !rticaria &ityriasis rosea &soriasis cabies :ichen plan!s is a common inflammatory disease of the skin characterised by violaceo!s/ polygonal/ pr!ritic pap!les. *t commonly affects the wrists and ankles. &ap!lar !rticaria presents as chronic or rec!rrent erythemato!s pap!les ca!sed by a hypersensitivity reaction to insect bites. &ityriasis rosea presents as ann!lar pap!los#!amo!s pla#!es with a peripheral collarette of scales. &soriasis presents as pap!los#!amo!s pla#!es over the e2tensors with micaceo!s scaling. cabies presents with itchy pap!les and b!rrows in fle2ors and web spaces of the hands and feet. ---------------------------------------Question: 29 of 148 Time taken: 01:01:08

37

5 middle aged male with diabetes presented with a tender lesion %as shown' over his b!ttock of two days@ d!ration. "hat is the most likely diagnosis$ %&lease select 1 option' )arb!ncle )orrect)orrect .cthyma 6ollic!litis *mpetigo ycosis 5 carb!ncle is a confl!ent collection of f!r!ncles which res!lts in a painf!l ind!rated swelling which is tender and discharges p!s from m!ltiple follic!lar orifices. .cthyma !s!ally follows trivial tra!ma s!ch as a scratch or insect bite on the legs and develops into a small p!st!lar lesion on an erythemato!s base/ with an adherent hard cr!st of dried e2!date below which !lceration e2ists. 6ollic!litis presents with erythemato!s painf!l follic!lar p!st!les. !perficial infection of the skin ca!sed by taphylococc!s a!re!s and treptococc!s pyogenes is common in children. *mpetigo classically involves 33

the epidermis and presents with honey colo!red cr!sts. *t is contagio!s and spreads among contacts as well as by a!toinoc!lation. ycosis barbae presents clinically with inflammatory follic!lo-centric p!st!les commonly on the glabro!s areas of the skin. ----------------------------------------Question: 30 of 148 Time taken: 01:01:5?

5n elderly male presented with dystrophic nail plates as shown along with an itchy rash of si2 months@ d!ration on the adAacent areas of the fingers and both groins. "hat is the most likely diagnosis$ %&lease select 1 option' )hronic &aronychia *ncorrect*ncorrect answer selected *rritant contact dermatitis +nychomycosis This is the correct anwserThis is the correct answer &achyonychia congenita &soriatic nail dystrophy +nychomycosis is a f!ngal infection of the nails. *t may involve the entire nail plate or a part of it with dystrophy of the nail plate and a moth eaten appearance. *t is not !ncommon to have the adAacent skin of the fingers/ toes/ hands and feet affected by the f!ngal infection.

39

)hronic paronychia presents with itchy painf!l swelling of the nail folds along with dystrophy of the nail plate. )hronic e2pos!re to irritants can lead to irreg!lar b!ckling and dystrophy of the nail plate. &achyonychia congenita presents with dystrophic/ thickened nails at or soon after birth along with palmoplantar keratoderma. &soriatic nail dystrophy presents with dystrophy of the nail plate/ s!b!ng!al hyperkeratosis/ oil spots/ onycholysis and splinter haemorrhages. 5ltho!gh it can involve the fle2!res s!ch as the groin the rash is not itchy. ,ermatophyte f!ngal infection of the nail presents with ,ystrophy of the affected nails. 5 moth eaten appearance as evident in the middle finger is characteristic. )ommonly the affected individ!als have evidence of dermatophytosis elsewhere/ s!ch as Tinea pedis/ Tinea corporis/ Tinea cr!ris %as in this case'. ----------------------------------------------------Question: 31 of 148 Time taken: 01:53:73

5 yo!ng adolescent girl presented with a rash in both a2illary regions occ!rring a day after application of a new deodorant. 35

"hat is the most likely diagnosis$ %&lease select 1 option' .rythrasma *ncorrect*ncorrect answer selected (erpes zoster *mpetigo *ntertrigo *rritant contact dermatitis This is the correct anwserThis is the correct answer *rritant contact dermatitis presents with vesic!lar er!ption at sites of contact with the irritant s!ch as deodorants/ often on the e2posed areas s!ch as e2tremities or any other affected area s!ch as the a2illary regions in this case. 5 b!rning sensation is common as opposed to the itching e2perienced in allergic contact dermatitis. .rythrasma appears as reddish-brown slightly scaly patches with sharp borders. The patches occ!r in moist areas s!ch as the groin/ armpit/ and skin folds. *t is an infective condition ca!sed by )orynebacteri!m spp. (erpes zoster presents as gro!ped vesic!lar er!ptions on an erythemato!s base in a dermatomal distrib!tion. *mpetigo commonly presents in children with painf!l b!llo!s lesions with honey colo!red cr!sts. *ntertrigo is an inflammatory disorder of the skin affecting the folds commonly in the obese and diabetics and may become secondarily infected. -------------------------------Question: 32 of 148 Time taken: 01:54:53

3?

5 middle aged male presented with dystrophic nail plates as shown with an itchy rash of si2 months@ d!ration in both groins. "hat is the most likely diagnosis$ %&lease select 1 option' )hronic paronychia *ncorrect*ncorrect answer selected *rritant contact dermatitis +nychomycosis This is the correct anwserThis is the correct answer &achyonychia congenita &soriatic nail dystrophy +nychomycosis is a f!ngal infection of the nails. *t may involve the entire nail plate or a part of it with dystrophy of the nail plate and a moth eaten appearance. )hronic paronychia presents with itchy painf!l swelling of the nail folds with dystrophy of the nail plate. )hronic e2pos!re to irritants can lead to irreg!lar b!ckling and dystrophy of the nail plate. &achyonychia congenita presents with dystrophic/ thickened nails at or soon after birth along with palmoplantar keratoderma. 34

&soriatic nail dystrophy presents with dystrophy of the nail plate/ s!b!ng!al hyperkeratosis/ oil spots/ onycholysis and splinter haemorrhages. 5ltho!gh it can involve the fle2!res s!ch as the groin the rash is not itchy. ----------------------------------------------Question: 33 of 148 Time taken: 01:53:93

5 middle aged male with a known case of bronchial asthma presented with rec!rrent episodes of itching/ vesic!lation and oozing over the c!bital fossae and popliteal fossae. "hat is his diagnosis$ %&lease select 1 option' 5topic dermatitis )orrect)orrect ,ermatitis herpetiformis *mpetigo *rritant contact dermatitis &emphig!s v!lgaris 5topic dermatitis is an imm!ne mediated disorder with onset in infancy. *t typically presents on the face in infancy followed by involvement of e2tensors in early childhood and fle2ors in ad!lthood. These patients often have associated atopic disorders s!ch as bronchial asthma. ,ermatitis herpetiformis is an a!toimm!ne blistering disorder associated with a gl!ten-sensitive enteropathy. ,ermatitis herpetiformis is characterised by 33

e2tensor involvement with gro!ped e2coriations/ pap!lovesic!lar er!ptions and !rticarial lesions. *mpetigo presents in children with painf!l b!llo!s lesions with honey colo!red cr!sts. *rritant contact dermatitis presents with vesic!lar er!ption at sites of contact with the irritant/ often on the e2posed areas s!ch as e2tremities. 5 b!rning sensation is common as opposed to the itching e2perienced in allergic contact dermatitis. &emphig!s v!lgaris presents with vesic!lob!llo!s lesions and erosions on normal-appearing skin. ----------------------------------------Question: 34 of 148 Time taken: 01:58:33

5 middle aged female patient presented an anaesthetic pla#!e on the face of three months d!ration. "hat is the most likely diagnosis$ %&lease select 1 option' 1ran!loma ann!lare *ncorrect*ncorrect answer selected (ansen@s disease This is the correct anwserThis is the correct answer arcoidosis Tertiary syphilis Frticaria

38

(ansen@s disease/ more commonly known as leprosy/ is the correct option here. *t classically prod!ces reddish patches or hypopigmented areas of skin/ with red!ced sensation. These are re#!ired for the disease to be diagnosed. T!berc!loid (ansen@s prod!ces anaesthetic pla#!es. :oss of sensation is !ncommon even in s!ch lesions on the face beca!se of rich innervation of the skin here. Th!s anaesthesia in a pla#!e on the face has no differential diagnosis e2cept (ansen@s disease. The localised form of gran!loma ann!lare presents with ann!lar or arc!ate pla#!es. The margin may be contin!o!s or consist of discrete pap!les coalescing together. ensation is preserved. arcoidosis may present with ann!lar/ arc!ate lesions or pap!los#!amo!s lesions. ensation is preserved. Tertiary syphilis presents with nod!lar syphilide/ pap!los#!amo!s lesions or g!mma. ensation is preserved. Frticaria presents with rec!rrent wheals which !s!ally s!bside in 79 ho!rs. There is no sensory loss. ---------------------------------------Question: 35 of 148 Time taken: 07:?0:11

90

5 yo!ng female patient presented with the above rash of few ho!rs d!ration after tending her garden. he had a similar milder rash on the arms a few weeks ago. "hat is the most likely diagnosis$ %&lease select 1 option' 5llergic contact dermatitis )orrect)orrect 6ollic!litis (erpes zoster *rritant contact dermatitis &aeder!s dermatitis 6ollic!litis presents with painf!l pap!lop!st!lar follic!lar lesions. (erpes zoster presents with ac!te onset gro!ped vesic!lar er!ptions in dermatomal distrib!tion. *rritant contact dermatitis presents with vesic!lar er!ption at sites of contact with the irritant/ often on the e2posed areas s!ch as e2tremities. &aeder!s dermatitis is an ac!te irritant contact dermatitis which presents as pla#!es/ linear streaks or @kissing@ lesions at sites where the offending pederin from the cr!shed beetle comes into contact with the skin. The #!estion mentions a past history of similar complaints a few days back. 5llergic contact dermatitis needs a sensitization dose following which the s!bse#!ent e2pos!res manifest early with severe manifestations. *rritant contact dermatitis does not need a sensitization dose and manifests almost immediately after e2pos!re. This case had a milder sensitization e2pos!re earlier followed by a severe reaction on the s!bse#!ent e2pos!re. (ence it is a case of 5llergic contact dermatitis. ---------------------------------------------Question: 36 of 148 Time taken: 07:?0:97 91

5 yo!ng child with a known case of plain warts on the face developed the shown linear distrib!tion of the lesions. "hat is this phenomenon called$ %&lease select 1 option' 5!spitz@ sign *ncorrect*ncorrect answer selected )arpet tack sign 1orlin sign Doebner phenomenon This is the correct anwserThis is the correct answer -ayna!d@s phenomenon Doebner phenomenon refers to appearance of isomorphic skin lesions appearing along lines of tra!ma. The Doebner phenomenon may res!lt from either a linear e2pos!re or irritation. *t is observed in conditions s!ch as vitiligo/ lichen plan!s/ psoriasis/ moll!sc!m contagios!m and warts. 5!spitz@ sign is seen in psoriasis where the lesion !pon scraping with a glass slide reveals pin-point bleeding on acco!nt of s!prapapillary thinning of the epidermis. )arpet tack sign: *n discoid l!p!s erythematos!s the removal of scale reveals m!ltiple horny pl!gs on the !nder s!rface of the scale giving the appearance of carpet tacks.

97

1orlin sign is the ability to to!ch the tip of the nose with the tong!e seen in .hlers-,anlos syndrome and some in the general pop!lation. -ayna!d@s phenomenon is a vasospastic disorder characterised by vasospasms that decrease blood s!pply to the respective regions !s!ally the hands and feet and infre#!ently the ear lobes/ lips and nose. )old is the classic trigger of the phenomenon. ----------------------------------------------Question: 37 of 148 Time taken: 07:47:98

5 3-month-old infant presented to the dermatologist with a red lesion over the left side of the face. *t was flat initially at birth and had progressively become elevated and boggy. "hat is the most likely diagnosis$ %&lease select 1 option' 5plasia c!tis *ncorrect*ncorrect answer selected )ystic hygroma (aemangioma This is the correct anwserThis is the correct answer :ymphangioma #!amo!s cell carcinoma 93

(aemangiomas may appear at birth as s!btle/ faint telangiectatic patch %the newest classification of infantile haemangiomas have removed the term @caverno!s haemangiomas@'. The lesion progresses variably over the first 5-? months of life/ becoming more palpable and more e2tensive. The overall clinical appearance is dependent on the depth of lesion. 5plasia c!tis presents with congenital absence of skin. )ystic hygroma is a congenital m!ltiloc!lated lymphatic lesion that is classically fo!nd in the left posterior triangle of the neck. This is the most common form of lymphangioma which typically transill!minates and is bl!ish in colo!r. :ymphangioma of the head and neck region involves swelling or mass that is soft to palpation and well circ!mscribed or diff!se. *t may be well to illdefined and is often associated with a bl!ish discoloration. ee cystic hygroma above. #!amo!s cell carcinoma most commonly presents in the middle aged and elderly as keratotic pla#!es in the s!n e2posed areas. ----------------------------------Question: 38 of 148 Time taken: 07:104:71

99

5 5-year-old boy was bro!ght with pigmentation aro!nd his mo!th and oral m!cosa of si2 months d!ration. "hat is the most likely diagnosis$ %&lease select 1 option' 5' )ondylomata lata *ncorrect*ncorrect answer selected B' )ongenital melanocytic naev!s )' 6reckles ,' C!cosal lichen plan!s .' &e!tz-Gegher@s syndrome This is the correct anwserThis is the correct answer +ption 5: )ondylomata lata - in secondary syphilis/ condylomata lata appear as flat-topped warty pap!les/ affecting the m!cosa. +ral m!cosa are rarely affected. +ption B: )ongenital melanocytic naev!s presents with naevi of varying sizes at birth. +ption ): 6reckles present as brownish mac!les on s!n e2posed sites and do not affect the m!cosa.

95

+ption ,: C!cosal lichen plan!s commonly presents as bilateral/ white lesions in the b!ccal and ling!al m!cosa. They may be retic!lar/ pap!lar or pla#!e-like or may have a violaceo!s h!e. C!coc!taneo!s pigmentation and hyperpigmented mac!les typically occ!r in &e!tz-Geghers syndrome. They appear as small brown or dark bl!e spots as lentigenes/ most commonly in the perib!ccal area and b!ccal m!cosa. The pigmentation in perioral region often crosses the vermillion border and can e2tend to the perinasal area too. &e!tz Geghers syndrome is associated with gastrointestinal polyposis. -------------------------------------------Question: 39 of 148 Time taken: 07:103:13

5 1-month-old otherwise healthy baby presented with a rash on the scalp with greasy scaling. "hat is the most likely diagnosis$ %&lease select 1 option' 5llergic contact dermatitis *ncorrect*ncorrect answer selected )radle cap This is the correct anwserThis is the correct answer 6ollic!litis *mpetigo 9?

Tinea capitis )radle cap presents with yellow/ greasy/ scaly and cr!sted lesions that occ!r on the scalp of recently born babies. )radle cap most commonly begins sometime in the first three months. 5llergic contact dermatitis will present with itchy pap!lovesic!lar er!ption at the site of contact of the allergen. 6ollic!litis presents with obvio!s follic!lar involvement in a case of painf!l pap!lop!st!lar lesions. *t is commoner in glabro!s areas of the skin. *mpetigo/ a s!perficial infection of the skin ca!sed by taphylococc!s a!re!s and treptococc!s pyogenes is common in children. *mpetigo classically involves the epidermis and presents with honey colo!red cr!sts. Tinea capitis or ringworm of the scalp is a common condition affecting children and !ncommonly ad!lts. The appearance varies from a few d!ll grey/ broken-off hairs with a little scaling/ detectable only on caref!l inspection/ to a severe/ painf!l/ inflammatory mass covering most of the scalp. *tching is variable. --------------------------------------------------------------Question: 40 of 148 Time taken: 07:103:55

94

5 50-year-old male presented with b!llo!s lesions on both his feet. (e also had dystrophic nails and an itchy scaly rash affecting the groins and interdigital web spaces of his feet. "hat is the most likely diagnosis$ %&lease select 1 option' B!llo!s impetigo *ncorrect*ncorrect answer selected B!llo!s pemphigoid B!llo!s tinea pedis This is the correct anwserThis is the correct answer .rythema m!ltiforme (erpes zoster B!llo!s tinea pedis is a severe variant of tinea pedis ca!sed by T. mentagrophytes var.interdigitale affecting the entire sole and !ncommonly the foot dors!m. *t is !s!ally preceded by years of scaly rash of tinea pedis. .vidence of onychomycosis manifesting as dystrophic moth-eaten appearance of nail plates and dermatophytosis elsewhere s!ch as tinea cr!ris completes the pict!re. B!llo!s impetigo presents with painf!l b!llo!s lesions in children. B!llo!s pemphigoid commonly starts with itching and a non-specific rash on the limbs that may be either !rticaria-like or occasionally eczemato!s. *t 93

presents as erythemato!s and !rticated pla#!es followed by tense blisters over the affected areas of tr!nk/ limbs and fle2!res. .rythema m!ltiforme classically presents with target lesions with a central b!llo!s lesion s!rro!nded by an erythemato!s halo following the intake of an offending dr!g. (erpes zoster presents with vesicob!llo!s lesions in a dermatomal distrib!tion. ------------------------------------------------Question: 41 of 148 Time taken: 03:150:37

5 middle aged male presented with s!dden onset non-tender ann!lar erythemato!s pla#!es with p!st!lation. (e had been managed with systemic steroids in the recent past for an ac!te e2acerbation of bronchial asthma. (e gave a history of rec!rrent episodes of scaly pla#!es in the past. "hat is the most likely diagnosis$ %&lease select 1 option' 5llergic contact dermatitis *ncorrect*ncorrect answer selected 6ollic!litis &emphig!s v!lgaris &!st!lar psoriasis This is the correct anwserThis is the correct answer !bcorneal p!st!lar dermatosis

98

1eneralised p!st!lar psoriasis is known to precipitate in known psoriatics !pon withdrawal of systemic steroids administered for co-e2istent disabilities. The scaly pla#!es mentioned in the clinical description are old psoriasis lesions. 5llergic contact dermatitis will present with itchy pap!lo-vesic!lar er!ption at the site of contact of the allergen. 6ollic!litis presents with painf!l follic!locentric p!st!les. &emphig!s v!lgaris presents with vesic!lob!llo!s lesions and erosions on normal appearing skin. !bcorneal p!st!lar dermatosis occ!rs mainly in fle2!res and on the fle2or aspect of the limbs. *t presents with characteristic p!st!les/ with p!s which acc!m!lates in the lower half of a f!lly developed p!st!le/ leaving clear fl!id in the !pper half. &!st!les may coalesce/ forming ann!lar or serpigino!s patterns with a scaly edge. The er!ption fades to leave faint hyperpigmentation and s!ccessive s!ch episodes may occ!r. ------------------Question: 42 of 148 Time taken: 03:151:79

50

5 7-year-old child presented with m!ltiple %more than three' hypopigmented lesions on the anterior chest wall and tr!nk since the age of one year. &arents vol!nteered a history of rec!rrent seiz!res. .2amination also revealed few r!bbery pinkish pla#!es on the low back. "hat is the most likely diagnosis of the hypopigmented lesions$ %&lease select 1 option' 5sh leaf mac!le )orrect)orrect (ansen@s disease =ev!s depigmentos!s &ityriasis alba 0itiligo (ypomelanotic mac!les oval at one end and tapering at the other end are called ash leaf mac!les. The lesions are hypopigmented and not depigmented like vitiligo %ash leaf lesions may look depigmented/ similar to nev!s depigmentos!s'. Core than three s!ch lesions along with shagreen patch %the pink to brownish r!bbery pla#!es' among other feat!res satisfy the diagnostic criteria of t!bero!s sclerosis comple2 %T )'. kin manifestations ass!me importance in the diagnosis of T ). (ansen@s disease presents !ncommonly at this age. kin manifestations incl!de hypopigmented mac!les which may be hypo-anaesthetic to normoanaesthetic depending on the spectr!m of the disease/ along with pla#!es in the t!berc!loid pole which wo!ld be hypo-anaesthetic. =ev!s depigmentos!s and option ./ vitiligo/ present with depigmented lesions. &ityriasis alba: the individ!al lesions are ro!nded/ oval or irreg!lar hypopigmented mac!les which are !s!ally not well marginated. The lesions are often slightly erythemato!s and have fine scaling. The initial erythema s!bsides leaving only persistent fine scaling and hypopigmentation. 51

--------------------Question: 43 of 148 Time taken: 03:151:50

5 yo!ng adolescent male presented with m!ltiple lesions on the face. "hat is the most likely diagnosis$ %&lease select 1 option' 5llergic contact dermatitis *ncorrect*ncorrect answer selected )omedones This is the correct anwserThis is the correct answer =od!locystic acne ycosis barbae Tinea barbae )omedones are the first stage of acne v!lgaris common in adolescents. )omedones may be open %blackheads' or closed %whiteheads'. ebaceo!s openings filled with pl!gs of seb!m and slo!ghed-off cells res!lting from d!ctal hypercornification give rise to blackheads. 5llergic contact dermatitis will present with itchy pap!lo-vesic!lar er!ption at the site of contact of the allergen. =od!locystic acne presents with m!ltiple inflammatory lesions over the face. ycosis barbae presents clinically with inflammatory follic!locentric p!st!les.

57

Tinea barbae presents with itchy follic!lar lesions and ann!lar pla#!es on glabro!s areas of the face. ------------------------Question: 44 of 148 Time taken: 03:157:73

5 17-year-old child presents with the above depressed sclerotic linear lesion over the left frontoparietal scalp of five years d!ration. There is a history of initial progression and thereafter a static phase for the past two years. -adiographs and ne!roimaging reveal no abnormal findings. There is no history of seiz!res or any history of ne!rological involvement. "hat sho!ld be the first line of management$ %&lease select 1 option' (igh dose *0 methylprednisolone *ncorrect*ncorrect answer selected =o active management This is the correct anwserThis is the correct answer &rophylactic anti-epileptics ystemic corticosteroids Frgent s!rgery .n co!p de sabre/ a variant of scleroderma/ is characterized by a linear/ atrophic depression affecting the frontoparietal aspect of the face and scalp/ s!ggestive of a stroke from a sword/ as shown in the image. !ch lesions may e2tend into the !nderlying tiss!es. calp involvement res!lts in scarring alopecia.

53

This condition/ altho!gh !ncommon/ typically presents with an initial actively progressive phase followed by sclerosis and inactive b!rnt o!t stage. -apid progression may lead to !nderlying cortical involvement/ seiz!res and eye complications. *n cases where the disease is inactive/ no active intervention is needed e2cept for keeping the patient !nder follow !p. (igh dose *0 methylprednisolone and systemic corticosteroids are incorrect. .mergent meas!res in the absence of active disease progression or ne!rooc!lar complications are not indicated. &rophylactic anti-epileptics have no role as the case has had no history of seiz!res/ ne!roimaging is normal and the condition is not progressing. !rgery has no role presently. The management of this case is controversial. ome paediatric dermatologists may give a trial of methotre2ate/ as the disease may still be slowly progressing and treatment may prevent worsening cosmetic disfig!rement. ---------------------Question: 45 of 148 Time taken: 03:157:53

5 17-year-old child presents with the above depressed sclerotic linear lesion of two years d!ration with a history of initial progression and thereafter a static phase. 59

"hat is his diagnosis$ %&lease select 1 option' 5crodermatitis chronica atrophicans *ncorrect*ncorrect answer selected 5plasia c!tis .n co!p de sabre This is the correct anwserThis is the correct answer &ost tra!matic scar cleroedema .n co!p de sabre/ a variant of scleroedema is characterised by a linear/ atrophic depression affecting the frontoparietal aspect of the face and scalp/ s!ggestive of a stroke from a sword/ as shown in the image. !ch lesions may e2tend into the !nderlying tiss!es. calp involvement res!lts in scarring alopecia. 5crodermatitis chronica atrophicans affects the acral parts and not the scalp. 5plasia c!tis is congenital absence of skin. *t is present at birth. &ost tra!matic scar will present with obvio!s history of tra!ma in s!ch a case. carring will be evident while this child has sclerosis with no obvio!s scarring. cleroedema is a misnomer. *t presents with ind!ration and occasional erythema. The lesions reveal no sclerosis and altho!gh it can present over the face/ linear lesions s!ch as this are absent. ---------------------Question: 46 of 148 Time taken: 03:153:73

55

5 middle aged lady working in a resta!rant kitchen presented with swelling of her right th!mb of two months d!ration. "hat is the most likely diagnosis$ %&lease select 1 option' 5c!te paronychia *ncorrect*ncorrect answer selected )andidal paronychia This is the correct anwserThis is the correct answer ,ermatophytosis *ngrowing toe nail &!st!lar psoriasis )andidal paronychia presents commonly in those chronically e2posed to wet working conditions. The nail fold is !s!ally red and swollen with loss of c!ticle/ detachment of nail fold from the dorsal s!rface of the nail plate/ leading to pocketing. +ccasionally/ thick white p!s may discharge. 5c!te paronychia is an ac!te tender inflammatory swelling of the nail folds ca!sed by taphylococc!s a!re!s commonly. ,ermatophytosis presents with onychomycosis with a moth eaten appearance of the nail plate.

5?

*ngrowing toe nail presents with ac!te onset inflammation of the affected digit with the distal corner of the nail plate b!ried in the lateral nail fold. &!st!lar psoriasis variant acrodermatitis contin!a of (allopea! or parakeratosis p!st!losa affects the nail plate which may be lifted by sterile p!st!les with associated erythema and discomfort and s!bse#!ent nail loss. -------------------Question: 47 of 148 Time taken: 03:153:91

5 middle aged male farmer presented with m!ltiple itchy inflammatory lesions over the chin. (e vol!nteered a history of working at a cattle farm. "hat is the most likely diagnosis$ %&lease select 1 option' 5llergic contact dermatitis *ncorrect*ncorrect answer selected )arb!ncle 6ollic!litis Derion This is the correct anwserThis is the correct answer ycosis Cycotic sycosis or kerion of the beard is an inflammatory response to zoophilic organisms ca!sed commonly by Trichophyton verr!cos!m/ the organism responsible for cattle ringworm.

54

5llergic contact dermatitis presents with itchy vesic!lar er!ption at the site of contact with the allergen. )arb!ncle presents in a diabetic with a painf!l inflammatory swelling s!rmo!nted with m!ltiple draining sin!ses. 6ollic!litis presents with painf!l follic!lar p!st!les. ycosis presents as s!bac!te or chronic pyogenic infection involving the whole depth of the follicle. ------------------Question: 48 of 148 Time taken: 03:159:17

5n (*0 positive male presented with whitish discolo!ration of tong!e and oral m!cosa. "hat is the most likely diagnosis$ %&lease select 1 option' 5phtho!s stomatitis *ncorrect*ncorrect answer selected 1eographical tong!e (erpetic gingivostomatitis C!cosal candidiasis This is the correct anwserThis is the correct answer C!cosal lichen plan!s 53

*mm!nos!ppressed patients are prone to develop m!cosal candidiasis which manifests as a chalky white coating of the affected area. craping off the s!perficial coating may reveal an !nderlying raw s!rface. 5phtho!s stomatitis is also common in (*0 positive and manifests with rec!rrent !lceration of the oral m!cosa. 1eographical tong!e manifests as apparent bald areas of the tong!e. (erpetic gingivostomatitis presents as painf!l erosions of the oral m!cosa and gingiva. C!cosal lichen plan!s may present with white to violaceo!s mac!les and pla#!es over the oral m!cosa. The lesions cannot be scraped off. --------------------Question: 49 of 148 Time taken: 03:159:33

5 17-year-old child presents with an itchy rash aro!nd her ears which develops 79 to 93 ho!rs after wearing earings. he gives history of past episodes of similar complaints. 58

"hat is the most likely diagnosis$ %&lease select 1 option' 5llergic contact dermatitis )orrect)orrect 6ollic!litis (erpes zoster *rritant contact dermatitis &aeder!s dermatitis 5llergic contact dermatitis is a form of contact dermatitis which follows contact with the offending agent s!ch as nickel %commonly fo!nd in earrings'/ chemicals and plants. The initial sensitisation dose !s!ally res!lts in a mild episode. 6ollic!litis presents with painf!l pap!lop!st!lar follic!lar lesions. (erpes zoster presents with ac!te onset gro!ped vesic!lar er!ptions in dermatomal distrib!tion. *rritant contact dermatitis presents with vesic!lar er!ption at sites of contact with the irritant. The onset of er!ption in irritant contact dermatitis is s!dden/ compared to allergic contact dermatitis which typically develops over 79 to 93 ho!rs. 5 b!rning sensation rather than itching is the is the predominant symptom in irritant contact dermtitis. &aeder!s dermatitis is an ac!te irritant contact dermatitis which presents as pla#!es/ linear streaks or @kissing@ lesions at sites where the offending pederin from the cr!shed beetle comes into contact with the skin. ---------------------Question: 50 of 148 Time taken: 03:35:09

?0

5 yo!ng adolescent male presents with dystrophic nails/ with debris on the !nder s!rface of the nails and elevated distal end of nail plate. (e also had scaly pla#!es on his elbows and knees. "hat is the most likely diagnosis$ %&lease select 1 option' 5lopecia areata associated nail dystrophy *ncorrect*ncorrect answer selected *rritant contact dermatitis +nychomycosis &soriasis This is the correct anwserThis is the correct answer Trachyonychia &soriasis involves nails ca!sing dystrophy/ s!b!ng!al hyperkeratosis/ onycholysis/ splinter haemorrhages and oil spots. The scaly pla#!es in this case are s!ggestive of psoriatic pla#!es on the e2tensor s!rface of the limbs. 5lopecia areata prod!ces fine nail pits in parallel rows in a @tram track appearance@. )hronic irritant dermatitis of the hands present with inflammation of the skin typically manifested by erythema/ mild oedema/ and scaling in the peri!ng!al region with nail plate discolo!ration affecting m!ltiple fingers and th!mbs. +nychomycosis ca!ses nail dystrophy res!lting in the moth eaten appearance. Tinea corporis !s!ally ca!ses ann!lar scaly pla#!es which are itchy. ?1

Trachyonychia or 70-nail-dystrophy may be seen in lichen plan!s or alopecia areata. -------------------Question: 51 of 148 Time taken: 03:155:17

5 ?0-year-old male presents with a s!dden onset er!ption of blisters. (e gives a history of pain in the affected area a day prior to the er!ption of the blisters for which he applied a topical analgesic. "hat is the most likely diagnosis$ %&lease select 1 option' 5llergic contact dermatitis *ncorrect*ncorrect answer selected 6ollic!litis (erpes zoster This is the correct anwserThis is the correct answer *rritant contact dermatitis &aeder!s dermatitis (erpes zoster !s!ally affects after the age of 50 years as well as in the imm!nocompromised. *t typically presents with pain in the affected area in the pre-er!ptive phase followed shortly by er!ption of gro!ped vesic!lob!llo!s lesions in dermatomal distrib!tion. The other differentials mentioned will not be dermatomal in distrib!tionH however may present with

?7

blistering/ itching and b!rning in the affected area in the case of contact dermatitis/ and pain in follic!litis. 5llergic contact dermatitis presents with itchy pap!lo-vesic!lar er!ption at the site of contact of the allergen. 6ollic!litis presents with painf!l pap!lop!st!lar follic!lar lesions. *rritant contact dermatitis presents with vesic!lar er!ption at sites of contact with the irritant/ often on the e2posed areas s!ch as e2tremities. 5 b!rning sensation is e2perienced in irritant contact dermatitis as opposed to the itching in allergic contact dermatitis. &aeder!s dermatitis is an ac!te irritant contact dermatitis which presents as pla#!es/ linear streaks or @kissing@ lesions at sites where the offending pederin from the cr!shed beetle comes into contact with the skin. -----------------Question: 52 of 148 Time taken: 03:15?:03

5 mother bro!ght her three children aged less than 10 years/ all with the complaints of a rash aro!nd mo!th and nose of 10 days d!ration. The yo!ngest child was affected first following which the other two children were affected. The lesions have been rapidly spreading in the affected area. .2amination revealed 1-3 cms cr!sted erosions with golden yellow cr!sts aro!nd the lips and nares in all three children. The yo!ngest child also had similar lesions on his hands. ?3

"hat is the most likely diagnosis$ %&lease select 1 option' .cthyma *ncorrect*ncorrect answer selected (erpes simple2 *mpetigo This is the correct anwserThis is the correct answer &erioral dermatitis "arts !perficial infection of the skin ca!sed by taphylococc!s a!re!s and treptococc!s pyogenes is common in children. *mpetigo classically involves the epidermis and presents with honey colo!red cr!sts. *t is contagio!s and spreads among contacts as well as by a!toinoc!lation. .cthyma !s!ally follows trivial tra!ma s!ch as a scratch or insect bite on the legs and develops into a small p!st!lar lesion on an erythemato!s base/ with an adherent hard cr!st of dried e2!date below which !lceration e2ists. (erpes simple2 infection presents with rec!rrent gro!ped vesic!lar er!ptions on an erythemato!s base at m!coc!taneo!s A!nctions. &erioral dermatitis presents !s!ally in women as vesicop!st!les aro!nd the mo!th and rarely aro!nd the eyes/ nose/ or forehead. The rash appears very similar to acne. "arts present with verr!co!s pla#!es and pap!les more commonly over e2tremities. ---------------------Question: 53 of 148 Time taken: 03:15?:11

?9

5 yo!ng male athlete presented with pedal hyperhidrosis with a non-itchy rash on both soles/ with fo!l odo!r. .2amination revealed well sc!lpted painless pits of varying depths along both soles symmetrically. "hat is the most likely diagnosis$ %&lease select 1 option' orns *ncorrect*ncorrect answer selected .rythrasma &itted keratolysis This is the correct anwserThis is the correct answer &lantar warts Tinea pedis &itted keratolysis is a common infection of the thickly keratinised areas of the plantar soles by Cicrococc!s sedantari!s. *t presents with sharply defined pits in the thick skin of the plantar s!rface of feet in areas which stay in contact with footwear in yo!ng ad!lts. *t is !s!ally associated with pedal hyperhidrosis which promotes maceration and hence facilitates bacterial penetration. The lesions are !s!ally non-tender. &lantar warts are !s!ally painf!l/ however they may present as a mosaic collection which may be non-tender and tinea pedis presents with itchy pla#!es classically over the instep of the soles in an asymmetric fashion. )orns are painf!l keratotic lesions which develop at press!re points. ?5

.rythrasma appears as reddish-brown slightly scaly patches with sharp borders. The patches occ!r in moist areas s!ch as the groin/ armpit/ and skin folds. &lantar warts present with solitary or m!ltiple painf!l verr!co!s lesions on the soles with obvio!s loss of skin markings over the lesions. Tinea pedis presents with itchy scaly pla#!es typically over instep of soles and between toes. -------------------------->!estion: 59 of 193 Time taken: 03:3?:33 5 5-year-old boy was bro!ght in with crops of asymptomatic rash over the tr!nk of two months@ d!ration. .2amination revealed skin colo!red to pearly white and hemispherical to !mbilicated pap!lar lesions. .ach one is appro2imately 9 mm in diameter and there are appro2imately 70 of these lesions present. "hat is the most likely diagnosis$ %&lease select 1 option' )!taneo!s cryptococcosis *ncorrect*ncorrect answer selected 6ollic!litis (erpes simple2 Coll!sc!m contagios!m This is the correct anwserThis is the correct answer "arts The diagnosis of moll!sc!m contagios!m is obvio!s when a child presents with pearly white hemispherical lesions partic!larly if they are !mbilicated over limbs/ tr!nk or face in vario!s stages of evol!tion. ??

(*0 positive patients may present with cryptococcosis. )ryptococc!s neoformans infection affects 5-10I of patients with 5*, in the FD and F 5 and 30-90I in 5frica. Fp to 70I of patients with disseminated disease may have skin involvement. *n (*0< 5*, cryptococcal skin involvement sho!ld be s!spected when pap!lonod!lar necrotising skin lesions with central !mbilication/ like moll!sc!m contagios!m are enco!ntered in s!ch patients along with p!lmonary or ne!rological disease. (ence c!taneo!s cryptococcosis m!st be kept as a differential in a case of !mbilicated lesions on the skin. 6ollic!litis presents with painf!l pap!lop!st!lar follic!lar lesions. (erpes simple2 infection presents with rec!rrent gro!ped vesic!lar er!ptions on an erythemato!s base at m!coc!taneo!s A!nctions. "arts present with verr!co!s pla#!es and pap!les more commonly over e2tremities. ------------------------Question: 55 of 148 Time taken: 09:130:09 5 yo!ng 3-month-old presented with a rash of two days@ d!ration along lips/ nose and adAacent right cheek.

?4

The mother gave history of a short febrile illness preceding the rash. &ast history of similar complaints was present. "hat is the most likely diagnosis$ %&lease select 1 option' 5llergic contact dermatitis *ncorrect*ncorrect answer selected 6ollic!litis (and/ foot and mo!th disease (erpes simple2 This is the correct anwserThis is the correct answer *mpetigo (erpes imple2 vir!s infection is common in children and adolescents and is ca!sed by two closely related vir!s types/ type 1 and type 7. *t commonly presents with rec!rrent painless gro!ped vesicles aro!nd m!coc!taneo!s A!nctions s!ch as the mo!th and the nose. The first episode may be painf!l. !bse#!ent episodes present with mild discomfort with a history for per-er!ption a!ra over the affected area. Canagement is largely symptomatic in rec!rrent episodes and topical 5I aciclovir cream may red!ce discomfort. The case presented here reveals perinasal and perioral involvement typical of herpes simple2. econdary impetiginization may be evident in some cases. 5llergic contact dermatitis will present with itchy pap!lo-vesic!lar er!ption at the site of contact of the allergen. 6ollic!lar involvement is evident in a case of painf!l pap!lop!st!lar lesions. *t is more common in glabro!s areas of the skin. (and/ foot and mo!th disease presents with vesic!lar and b!llo!s lesions over palms/ soles/ b!ttocks and oral erosions. (istory of similar complaints may be present in contacts. -------------------------------------------Question: 56 of 148 Time taken: 09:130:38 ?3

5 71-year-old soldier presented with any itchy rash of two weeks@ d!ration affecting his hands/ feet/ groins and b!ttocks. The rash was typically more itchy at night. The patient gave a history of his dormitory colleag!es s!ffering from similar complaints. .2amination revealed n!mero!s e2coriated pap!les and b!rrows primarily localised to the web spaces of hands and feet as well as the natal cleft. "hat is the most likely diagnosis$ %&lease select 1 option' 5llergic contact dermatitis *ncorrect*ncorrect answer selected ,ermatophytosis 6ollic!litis *mpetigo cabies This is the correct anwserThis is the correct answer cabies is common in children and yo!ng ad!lts/ occ!rring more commonly in places where crowding facilitates transmission of the mite. The rash ca!ses itching more d!ring the nights when the patient is warmer. 5reas of skin scarce in pilosebaceo!s follic!les are the preferred site by the scabies mite. 5llergic contact dermatitis will present with itchy pap!lo-vesic!lar er!ption at the site of contact of the allergen.

?8

,ermatophytosis !s!ally presents with ann!lar scaly pla#!es with active peripheral margins and central clearing. The lesions are typically itchy and increase in n!mber over a period of time. 6ollic!litis presents with erythemato!s follic!lar pap!les and p!st!les. *mpetigo is a s!perficial infection of the skin common in children. *t classically involves the epidermis and presents with honey colo!red cr!sts. *t is contagio!s and spreads among contacts as well as by a!toinoc!lation. ---------------------------------Question: 57 of 148 Time taken: 09:133:18

5 30-year-old farmer from a tropical co!ntry presented with painless swelling of his foot of si2 months d!ration. (e vol!nteered a history of a trivial tra!ma while working his fields eight to nine months back. 6ollowing the inA!ry he noticed a grad!ally progressive painless swelling in the mid sole which s!bse#!ently discharged black grains from sin!ses which developed over the swelling. &lain radiograph revealed soft tiss!e swelling with a well preserved skeletal architect!re. "hat is the most likely diagnosis$ %&lease select 1 option' 6ollic!litis *ncorrect*ncorrect answer selected 40

*mpetigo :!p!s v!lgaris Cycetoma foot This is the correct anwserThis is the correct answer +steomyelitis Cycetoma foot ca!sed by Cad!rella mycetomatis follows tra!matic inoc!lation of the f!ng!s mostly on the foot and the lower leg. The earliest stage is a firm/ painless nod!le b!t/ with time/ pap!les/ p!st!les which break down to form draining sin!ses that appear on the skin s!rface following which the affected area becomes hard and swollen !s!ally witho!t significant pain e2tension to !nderlying bones and Aoints gives rise to periostitis/ osteomyelitis and arthritis. 5dvanced cases may present with destr!ction of bone within an infected area giving rise to gross deformities. There are !s!ally m!ltiple sin!s tracts draining p!s and gran!les. :ymph node involvement is rare. 6ollic!litis and impetigo do not occ!r on the sole. :!p!s v!lgaris !s!ally presents with an indolent grad!ally progressive pla#!e with periods of activity and remission with atrophic scarring. +steomyelitis will reveal painf!l swelling/ draining sin!ses and skeletal involvement on plain radiograph. --------------------------------------------Question: 58 of 148 Time taken: 09:13?:9?

41

5 30-year-old mother with her ?-year-old da!ghter presents with itching of the scalp with hair loss of one month d!ration. .2amination revealed patches of partial alopecia/ sharply c!t off circ!lar in shape/ with n!mero!s broken-off/ d!ll grey hairs in the alopecic patches. "ood@s lamp e2amination revealed green fl!orescence. "hat is the most likely diagnosis$ %&lease select 1 option' 5lopecia areata *ncorrect*ncorrect answer selected eborrhoeic dermatitis econdary syphilis Tinea capitis This is the correct anwserThis is the correct answer Trichotillomania Tinea capitis or ringworm of the scalp is a common condition affecting children and !ncommonly ad!lts/ where the ad!lts are !s!ally secondarily infected. The response to this infection is variable/ depending on the type of hair invasion/ the level of host resistance and the degree of inflammatory host response. The appearance therefore may vary from a few d!ll grey/ brokenoff hairs with a little scaling/ detectable only on caref!l inspection/ to a severe/ painf!l/ inflammatory mass covering most of the scalp. *tching is variable. haring of combs facilitates spread in the family.

47

5lopecia areata presents as non-itchy areas of hair loss with e2clamation mark hairs. eborrhoeic dermatitis presents with diff!se greasy scaling. (air loss in s!ch localised patches is not a feat!re. econdary syphilis presents with moth-eaten alopecia. Trichotillomania presents as patchy hair loss with hair of varying lengths in different as well as the same patch. *nvariably there is a history of a family member having observed serial pl!cking of hair by the patient. ------------------------------Question: 59 of 148 Time taken: 09:134:53

43

5 90-year-old male presented with progressive discolo!ration of his great toe nail/ which initially affected the distal part of the nail plate and progressively involved the more pro2imal parts of the same nail over a period of three months. (e also noticed a rash in the groins which had appeared appro2imately fo!r weeks back. .2amination revealed a moth-eaten appearance of the right great toe nail with a scaly ann!lar rash with an active peripheral margin in both groins. "hat is the most likely diagnosis$ %&lease select 1 option' 5llergic contact dermatitis *ncorrect*ncorrect answer selected 6i2ed dr!g er!ption +nychomycosis with tinea cr!ris This is the correct anwserThis is the correct answer &soriasis -eiter@s disease +nychomycosis is not !ncommon in males. *t typically affects the distal part of the nail plate with progressive involvement of the pro2imal portions of the nail. The great toe may be the only affected nail/ altho!gh it is not !ncommon to find other toe nails also affected over a period of time. The affliction of the great toe may be long standing and asymptomatic e2cept for the disfig!rement of the nail plate. The patient fre#!ently cons!lts the doctor when he or she develops other symptoms of dermatophytosis as a res!lt of spread of the infection commonly by a!to inoc!lation in areas s!ch as the groins/ tr!nk/ e2tremities or the scalp. 5llergic contact dermatitis will present with itchy pap!lo-vesic!lar er!ption at the site of contact of the allergen. 6i2ed dr!g er!ptions present with mac!les/ erythemato!s or hyperpigmented or b!llo!s lesions. (istory of dr!g intake may be forthcoming. +nset is

49

s!dden with rapid progression. :esions are !ncommonly scaly pla#!es which will not show typical peripheral activity of dermatophytosis. &soriasis presents with pap!los#!amo!s lesions with micaceo!s scaling. 5nn!lar lesions may be present. *tching is not a feat!re. -eiter@s disease typically presents with pap!los#!amo!s pla#!es/ keratoderma blennorrhagica and circinate balanitis which are not itchy. ------------------------->!estion: ?0 of 193 Time taken: 09:03:99 5 father/ 93-year-old/ and his son/ 70-year-old/ presented with m!ltiple itchy red lesions over their tr!nks and groins. The son had developed the rash initially after working o!t in the local gym following which his father noticed a similar rash afflicting him. "hat is the most likely diagnosis$ %&lease select 1 option' 5llergic contact dermatitis *ncorrect*ncorrect answer selected 6i2ed dr!g er!ption &soriasis arcoidosis Tinea corporis This is the correct anwserThis is the correct answer ,ermatophytosis is common in yo!ng ad!lts. *t !s!ally presents with ann!lar scaly pla#!es with active peripheral margins and central clearing res!lting in increasing size of the lesions. The lesions are typically itchy and increase in n!mber over a period of time. The affliction of legs/ thighs is termed tinea corporis. *t is an occ!pational dermatoses in athletes and may be rec!rrent in them if d!e preca!tions are not taken. The condition is contagio!s and can 45

typically spread in immediate contacts s!ch as family members if treatment is delayed or preca!tions are not taken. 5llergic contact dermatitis will present with itchy pap!lovesic!lar er!ption at the site of contact of the allergen. 6i2ed dr!g er!ption presents with erythemato!s or hyperpigmented mac!les or patches/ sometimes with central b!llae. (istory of dr!g intake may be forthcoming and a history of other rec!rrent episodes at the same site may be present. &soriasis presents with pap!los#!amo!s lesions with micaceo!s scaling. 5nn!lar lesions may be present. *tching is not a feat!re. arcoidosis may present with scaly pla#!es/ however feat!res of peripheral activity and central clearing are !s!ally not seen and itching is not a feat!re. ---------------------------------------->!estion: ?1 of 193 Time taken: 09:03:97 5 yo!ng male athlete presented with m!ltiple itchy scaly lesions over his legs and thighs. (istorically he had had similar lesions in the past which had been treated with !nspecified local medications. +n e2amination he was fo!nd to have m!ltiple erythemato!s scaly pla#!es with a raised peripheral margin/ a clear centre with hyper-pigmentation. "hat is the most likely diagnosis$ %&lease select 1 option' 5llergic contact dermatitis *ncorrect*ncorrect answer selected 6i2ed dr!g er!ption &soriasis 4?

arcoidosis Tinea corporis This is the correct anwserThis is the correct answer ,ermatophytosis is common in athletes. *t !s!ally presents with ann!lar scaly pla#!es with active peripheral margins and central clearing res!lting in increasing size of the lesions. The lesions are typically itchy and increase in n!mber over a period of time. The affliction on legs and thighs is termed tinea corporis. *t is occ!pational dermatoses in athletes and may be rec!rrent in them if d!e preca!tions are not taken. 5llergic contact dermatitis will present with itchy pap!lo-vesic!lar er!ption at the site of contact of the allergen. 6i2ed dr!g er!ption presents with erythemato!s or hyperpigmented mac!les or patches/ sometimes with central b!llae. (istory of dr!g intake may be forthcoming and a history of rec!rrent s!ch episodes at the same site may be present. &soriasis presents with pap!los#!amo!s lesions with micaceo!s scaling. 5nn!lar lesions may be present. *tching is not a feat!re. arcoidosis may present with scaly pla#!es/ however feat!res of peripheral activity and central clearing are !s!ally not seen and itching is not a feat!re. ---------------------------------Question: 62 of 148 Time taken: 09:133:98

44

5 70-year-old male presents with a very itchy rash afflicting both groins and s!prap!bic region of fo!r weeks d!ration. +n e2amination he had m!ltiple ann!lar scaly pla#!es with spreading peripheral borders and central clearing. "hat is his diagnosis$ %&lease select 1 option' 5llergic contact dermatitis *ncorrect*ncorrect answer selected ,ermatophytosis This is the correct anwserThis is the correct answer 6i2ed dr!g er!ption &soriasis arcoidosis ,ermatophytosis is common in yo!ng active ad!lts. *t !s!ally presents with ann!lar scaly pla#!es with active peripheral margins and central clearing res!lting in increasing size of the lesions. The lesions are typically itchy and increase in n!mber over a period of time. The affliction of groins is referred to as @tinea cr!ris@ and involvement of limbs and tr!nk as @tinea corporis@. !ch patients can also be said to be s!ffering from @tinea cr!ris et corporis@. +ption 5: 5llergic contact dermatitis will present with itchy pap!lo-vesic!lar er!ption at the site of contact of the allergen +ption ): 6i2ed dr!g er!ption present with erythemato!s or hyperpigmented mac!les or patches/ sometimes with central b!llae. (istory of dr!g intake 43

may be forthcoming and history of rec!rrent s!ch episodes at the same site may be present. +ption ,: &soriasis presents with pap!los#!amo!s lesions with micaceo!s scaling. 5nn!lar lesions may be present. *tching is not !s!ally a prominent feat!re. +ption .: arcoidosis may present with scaly pla#!es/ however feat!res of peripheral activity and central clearing are !s!ally not seen and itching is not a feat!re. ---------------------------------->!estion: ?3 of 193 Time taken: 09:17:74 5 14-year-old man comes to the clinic. (e has recently ret!rned from a holiday to pain with his friends and is very disappointed with the #!ality of his tan/ as there appear to be large depigmented areas on the skin of his abdomen and on his back. (e says the areas are itchy. +n e2amination yo! confirm the depigmentation/ and there is s!perficial scaling over the areas. "hich of the following is the most appropriate treatment$ %&lease select 1 option' +ral antif!ngals *ncorrect*ncorrect answer selected +ral corticosteroids -eass!rance Topical antif!ngals This is the correct anwserThis is the correct answer Topical corticosteroids This patient has clinical findings which are typical of those seen in pityriasis versicolor/ ca!sed by a f!ng!s known as Calassezia f!rf!r. This ca!ses more 48

of a problem in hot weather/ is present on the skin of the tr!nk/ and leads to the depigmentation seen here. Topical antif!ngals are the treatment of choiceH if the depigmentation is e2tensive/ a systemic compo!nd like fl!conazole may be !sed. --------------------Question: 64 of 148 Time taken: 09:17:94 5 ?7-year-old woman presents with severe na!sea and lethargy a few days after beginning diclofenac and amo2icillin from her 1& for pain and a !rinary tract infection. he has no past history of note apart from hypertension for which she takes ramipril/ and she believes she inA!red her back lifting a wardrobe. +n e2amination her B& is 158<87 mm(g/ she has bilateral crackles on a!sc!ltation of the chest/ her p!lse is 38 and reg!lar. 5bdominal e2amination is !nremarkable. he has a widespread erythemato!s rash. *nvestigations show Hb 11.8 g<dl %13.5-13' 3.8 2 108<l WCC %9-11' .osinophilia PLT 703 2 108<l %150-900' a 138 mmol<l %135-19?' ! ?.1 mmol<l %3.5-5' C" 337 mmol<l %48-113' #"ine $"otein ;; %&oo' W(ite )e&&s "hich of the following is the most likely diagnosis$ %&lease select 1 option' 5c!te t!b!lar necrosis *ncorrect*ncorrect answer selected )h!rg- tra!ss syndrome *nterstitial nephritis This is the correct anwserThis is the correct answer Cembrano!s nephropathy &yelonephritis 30

The rapid onset of renal fail!re/ co!pled with a rash and eosinophilia is highly s!spicio!s of a diagnosis of interstitial nephritis as a res!lt of e2pos!re to non-steroidal or amo2icillin. "hilst eosinophilia is not !s!ally seen in interstitial nephritis as a res!lt of = 5*,s/ it is seen in penicillin-ind!ced interstitial nephritis. )T scanning and !ltraso!nd are the main imaging modalities/ with *0F being rarely !sed now. -enal biopsy is definitive in cases where the diagnosis is in do!bt. -esol!tion of renal fail!re !s!ally begins after the ca!sative agent is discontin!ed. ---------------------------Question: 65 of 148 Time taken: 09:17:55 5 95-year-old man is referred to the dermatology clinic/ with an intensely itchy/ red/ scaling rash which affects his scalp predominantly and is worse in spring and winter time. (e also has a patch on his chest and aro!nd his beard. +n e2amination he has a severe scalp rash with cr!sting and scaling of skin. *nvestigations show Haemo*&obin 13.1 g<dl %13.5-13' W(ite )e&& )ount 5.8 2 108<l %9-10' 8 P&ate&ets 187 2 10 <l %150-900' +o'ium 138 mmol<l %139-193' Potassium 9.3 mmol<l %3.5-5' C"eatinine 101 Jmol<l %?0-170' +)a&$ bio$s, (yperkeratosis/ acanthosis and focal spongiosis (e tells yo! he cares most abo!t his scalp. "hich of the following is the most appropriate first line treatment for him$ %&lease select 1 option' oal tar shampoo *ncorrect*ncorrect answer selected

31

Detoconazole shampoo This is the correct anwserThis is the correct answer +ral prednisolone Tacrolim!s ointment Topical betamethasone This man has seborrhoeic dermatitis which tends to affect hair bearing areas of skin with the scalp the worst affected. *t is tho!ght that/ at least in part/ the activity of activated T cells may be enhanced by an increased reservoir of Calassezia yeasts. 5s s!ch/ reg!lar washing of the hair with a ketoconazole based shampoo has been shown to red!ce the severity of the condition. Topical corticosteroids have been shown to hasten recovery/ b!t may be associated with a rebo!nd effect and rapid rec!rrence of the rash when they are withdrawn. )oal tar shampoo and topical tacrolim!s are alternate treatments. 5steatotic dermatitis typically presents in the elderly with pr!ritic/ 2erotic/ scaly skin typically over the shins b!t may occ!r over the back and hands. Tar based preparations are never !sed as they wo!ld aggravate the condition. -----------------Question: 66 of 148 Time taken: 09:13:09 5 31-year-old woman comes to the dermatology clinic complaining that a mole on her forearm has changed shape/ enlarging to nearly three #!arters of a centimetre in diameter/ and altho!gh it was previo!sly homogeneo!s in colo!r/ parts of it have now become a very dark black. he has no significant past medical history b!t admits to significant !se of tanning beds and having spent a few years living in )alifornia. *nvestigations show: Haemo*&obin 13.0 g<dl %13.5-13' 37

W(ite )e&& )ount 5.9 2 108<l %9-10' P&ate&ets 700 2 108<l %150-900' -+. 11 mm<hr %1-70' +o'ium 138 mmol<l %139-193' Potassium 9.7 mmol<l %3.5-5' C"eatinine 110 Jmol<l %?0-170' .2cision biopsy: 5 mm thick lesion/ no !lceration. "hich of the following feat!res is most associated with a poor prognosis in this patient$ %&lease select 1 option' ,epth of the melanoma lesion )orrect)orrect 6emale se2 (er age :ack of !lceration ignificant e2pos!re to s!n beds !rvival is strongly correlated with depth of melanoma at the point of diagnosis/ with lesions over 9 mm thick being associated with a partic!larly poor o!tcome. +ther predictors of a poor o!tcome incl!de increasing age and male se2. Flceration of the lesion also implies greater risk of metastases. 6or lesions over 9 mm thick with !lceration/ five year s!rvival is less than 50I. ---------------------------->!estion: ?4 of 193 Time taken: 09:13:03 5 33-year-old woman has erythemato!s pla#!es with hyperpigmentation at the edge and central hypopigmentation. erological tests are !nremarkable and the patient is diagnosed with discoid l!p!s. "hich of the following therape!tic strategies is most appropriate initially$ 33

%&lease select 1 option' )yclophosphamide *ncorrect*ncorrect answer selected ,apsone Cethotre2ate +ral prednisolone !n protective meas!res This is the correct anwserThis is the correct answer ,iscoid l!p!s is photosensitive and so avoiding the s!n with protective clothing and s!nscreens/ and changing behavio!r is important. Topical or intradermal steroids and hydro2ychloro#!ine are !sef!l therapies to prevent scarring and new lesions appearing. The latter are less effective if patients contin!e to smoke. *f these treatments are ineffective/ second line dr!gs s!ch as methotre2ate/ azathioprine/ or thalidomide may be !sed. ----------------------->!estion: ?3 of 193 Time taken: 09:13:07 5 15-year-old boy comes to the dermatology clinic as his parents are concerned abo!t some changes they have noticed in his skin. (e is from a travelling comm!nity/ and as s!ch his parents have previo!sly sh!nned medical services. +n e2amination yo! notice that he has a n!mber of facial and peri!ng!al fibromata. (e also has a n!mber of hypomelanotic areas %at least fo!r' on e2amination of his skin. Ko! also notice gingival fibromata and pitting of his tooth enamel on e2amination of his mo!th.

39

+n which chromosome is the abnormality associated with this disease likely to be fo!nd$ %&lease select 1 option' )hromosome 7 *ncorrect*ncorrect answer selected )hromosome ? )hromosome 8 This is the correct anwserThis is the correct answer )hromosome 11 )hromosome 17 This boy has t!bero!s sclerosis/ which is inherited in a!tosomal dominant fashion/ with responsible defects having been identified on both chromosome 8 and chromosome 1?. These chromosomes carry codes for hamartin and t!berin/ protein gene prod!cts which are responsible for reg!lation of cell growth. Cost of the t!mo!rs which are prod!ced in t!bero!s sclerosis are hamartomas/ and vario!s phenotypes of the disease occ!r/ with some parents of patients having m!ch more s!btle feat!res than those seen in their children ----------------------->!estion: ?8 of 193 Time taken: 09:13:79 5 n!rse presents with severe swelling aro!nd the mo!th following her l!nch. he was treated for s!spected anaphyla2is and on f!rther #!estioning she says that previo!sly she has had an itchy rash on her hands after wearing late2 gloves. "hich of the following foods is the most likely to have been in her l!nch to e2plain this reaction$ %&lease select 1 option' 35

5sparag!s *ncorrect*ncorrect answer selected Banana This is the correct anwserThis is the correct answer 1ooseberry Blackc!rrant tar fr!it :ate2 is one of the allergens which has been shown to have an association with hypersensitivity to some plant antigens - the oral allergy syndrome. This refers to an allergic reaction in response to eating certain foods tho!ght to contain proteins which cross-react with antigens seen in late2. The reaction is classically limited to the mo!th/ tong!e and throat. The list of possible agents ca!sing this hypersensitvity is e2tensive/ and incl!des banana/ avocado/ potato/ tomato/ kiwi/ chestn!t/ passion fr!i/ mango/ pineapple/ apple/ peach/ watermelon and many more. -eference: alcedo 1/ ,iaz-&erales 5/ anchez-Conge -. The role of plant panallergens in sensitization to nat!ral r!bber late2. Curr Opin Allergy Clin Immunol. 7001H1%7':144-33. ----------------------->!estion: 40 of 193 Time taken: 09:13:77 5 51-year-old male presents with a rash that has been present intermittently over the last two years. +n e2amination there is a symmetrical rash over the cheeks nose and chin with m!ltiple pap!les and p!st!les. "hat is the most appropriate therapy for this patient$

3?

%&lease select 1 option' 6l!clo2acillin *ncorrect*ncorrect answer selected (ydro2ychloro#!ine *sotretinoin +2ytetracycline This is the correct anwserThis is the correct answer &rednisolone The description is that of acne rosacea partic!larly in view of the distrib!tion/ d!ration and absence of any other feat!res. The most appropriate treatment is a tetracycline. ---------------------->!estion: 41 of 193 Time taken: 09:13:14 )oncerning ne!rofibromatosis type 1 %=61'/ which one of the following statements is tr!e$ %&lease select 1 option' Bilateral aco!stic ne!romas are common *ncorrect*ncorrect answer selected )linical severity in individ!als is similar in a given family =ew m!tations occ!r rarely &igmented spots on the iris are a characteristic feat!re This is the correct anwserThis is the correct answer The diagnosis is likely if two cafL a! lait patches are present :isch nod!les of the iris are present in more than 80I of patients. Bilateral aco!stic ne!romas are a hallmark feat!re of ne!rofibromatosis type 7.

34

.2pressivity of the gene is highly variable and members of the same family !s!ally show wide differences in clinical symptoms. =61 is one of the most common a!tosomal dominant conditions. (owever almost half of all cases give no family history and are new m!tations. The m!tation rate is estimated to be 1:10/000 gametes. The diagnosis is s!ggested by si2 or more cafL a! lait mac!les %spots'/ each over 5 mm in diameter in prep!bescent individ!als and over 15 mm in post p!bertal individ!als. --------------------->!estion: 47 of 193 Time taken: 09:13:73 "hich of the following is tr!e regarding diabetic foot !lceration$ %&lease select 1 option' - 5!tonomic ne!ropathy res!lts in red!ced peripheral blood flow *ncorrect*ncorrect answer selected - )all!s formation at press!re areas is an important predictor of !lceration This is the correct anwserThis is the correct answer - &lantar !lceration is most commonly d!e to atherosclerosis - kin infection is the most common initiating event in !lceration - -adiography can readily disting!ish between )harcot@s Aoint and osteomyelitis )all!s formation at press!re areas is an important predictor of potential !lceration. &lantar !lceration is !s!ally a conse#!ence of ne!ropathy. Cinor skin tra!ma is probably the most common initiating event.

33

t!dies have shown that a!tonomic ne!ropathy increases peripheral blood flow/ probably as a res!lt of red!ced arterioveno!s sh!nting. *t is diffic!lt to disting!ish radiographically between )harcot@s Aoint and osteomyelitis/ as neither have specific signs in their early stages. ----------------->!estion: 43 of 193 Time taken: 09:13:54 "hich of the following determines the primary mechanical properties of skin$ %&lease select 1 option' ,ermis *ncorrect*ncorrect answer selected trat!m basale trat!m corne!m This is the correct anwserThis is the correct answer trat!m gran!los!m !bc!taneo!s tiss!e The strat!m corne!m is the last layer and provides a mechanical barrier to the skin and therefore determines the mechanical f!nctions of the skin. The hands and feet have thick strat!m corne!m as compared to the lips and eyelids. The thicker the strat!m corne!m is the more protection there is for the skin. The dermis also has some factor to play with its elastic fibres and fibro!s tiss!e. The rest of the layers are also important b!t the mechanical properties are primarily determined by the strat!m corne!m. ------------------>!estion: 49 of 193 Time taken: 09:13:55 38

5 93-year-old female presents with a weepy/ erythemato!s rash mainly affecting the forehead/ scalp/ neck and !pper back. Three days earlier she had !sed red hair dye at home to self-administer @highlights@. "hat is the likely diagnosis$ %&lease select 1 option' 5cne rosacea *ncorrect*ncorrect answer selected )ontact allergic dermatitis This is the correct anwserThis is the correct answer :!p!s erythematos!s &soriasis eborrhoeic dermatitis (air dye contains s!bstances which may ind!ce an eczemato!s response in form of contact allergic dermatitis. This type of reaction is typical for this sort of time scale/ and is an e2ample of a type *0/ or delayed/ hypersensitivity reaction. ensitisation occ!rs on initial e2pos!re to the allergen and @memory@ T-cells proliferate in lymphoid tiss!e. !bse#!ent e2pos!re to allergen ind!ces activation of the T-lymphocytes and an inflammatory response. (airdressing chemicals are a very common ca!se of contact allergic dermatitis/ a disorder which is very common amongst the hairdressing comm!nity. 5 lady at this age is !nlikely to present with a new/ previo!sly !ndiagnosed case of eczema.

80

5cne rosacea is !s!ally a disorder of the skin on the cheeks and nose. 5c!te c!taneo!s l!p!s erythematos!s presents with an erythemato!s rash over the face and neck. (owever there are no eczemato!s feat!res. ---------------------->!estion: 45 of 193 Time taken: 09:19:04 ,!ring a follow !p visit at an asthma clinic a 33-year-old female complains of the appearance of a mole. "hich of the following characteristics of the lesion wo!ld raise s!spicion that it is a malignant melanoma$ %&lease select 1 option' :esion has irreg!lar o!tline )orrect)orrect :esion is deeply pigmented :esion is present on face :esion is raised :esion is ? mm in diameter The mnemonic of 5B),. regarding characteristics of a melanoma are as follows: 5 - 5symmetry - one half of the lesion does not match the other half B - Border irreg!larity ) - )olo!r variegation - pigmentation is not !niform , - ,iameter- a diameter 4 mm warrants investigation altho!gh changes in size are also important . - .vol!tion - evolving size or changes in characteristics s!ch as nod!les. --------------->!estion: 4? of 193 81

Time taken: 09:19:05 5 94-year-old woman presented with a history several years of dysphagia/ hard calcified nod!les in the fingers/ and cold hands. .2amination revealed calcified nod!les/ sclerodactyly and facial telangiectasia. "hich one of the following antibodies is most likely to be fo!nd in the blood$ %&lease select 1 option' 5nticardiolipin *ncorrect*ncorrect answer selected 5nticentromere This is the correct anwserThis is the correct answer 5nti-,=5 antibodies 5ntimitochondrial 5ntimyelopero2idase This patient has feat!res of )-. T syndrome/ making systemic sclerosis the most likely diagnosis. ystemic sclerosis is a chronic a!toimm!ne disease characterised by increased fibroblast activity and fibrosis in a n!mber of different organ systems. =inety to 85I of patients have positive antin!clear antibodies. There are two maAor s!btypes: M :imited c!taneo!s and M ,iff!se c!taneo!s. )-. T syndrome is an older term for the limited c!taneo!s form %calcinosis/ -ayna!ds@ phenomenon/ oeosophageal dysmotility/ sclerodactyly/ telangiectasia'. 87

&atients with systemic sclerosis can present with M kin abnormalities M C!sc!loskeletal changes M 1astrointestinal complications M &!lmonary disease M -enal crisis and M ,ry eyes and mo!th. 5 n!mber of a!toantibodies against e2tractable n!clear antigens can be detected in patients with systemic sclerosis. 5nticentromere antibodies and antitopoisomerase * antibodies are the classic a!toantibodies associated with the disease. 5nticentromere antibodies are linked with limited c!taneo!s involvement and isolated p!lmonary hypertension/ and a good prognosis/ whereas antitopoisomerase * is linked with diff!se skin disease and p!lmonary fibrosis and a higher mortality. 5dditional a!toantibodies which can be detected are anti-=5 polymerase/ antiF3-=&/ antiD! and antiF1-=&. 5nticardiolipin antibodies are seen in antiphospholipid syndrome. 5nti,=5 antibodies are associated with systemic l!p!s erythematos!s. 5ntimitochondrial antibodies are seen in primary bilary cirrhosis. 5ntimyelopero2idase antibodies are also referred to as p5=)5/ and can be seen in a variety of vasc!litides.

83

-eferences: 5!toantibody profiles in systemic sclerosis: predictive val!e for clinical eval!ation and prognosis. (amag!chi K et al. G ,ermatol 7010 GanH34%1':97-53 ------------------>!estion: 44 of 193 Time taken: 09:19:00 5 58-year-old patient of o!th 5sian origin presents with a widespread blistering rash. "hich of the following feat!res wo!ld be consistent with a diagnosis of pemphig!s$ %&lease select 1 option' 5canthosis *ncorrect*ncorrect answer selected Blisters arising within the s!bepidermal area *g5 antibodies +ral involvement This is the correct anwserThis is the correct answer Treatment with methotre2ate &emphig!s is associated with loss of intercell!lar cohesion in the lower part of the epidermis/ leading to acantholysis %separation of keratinocytes'. &emphig!s is classically associated with flaccid blistering/ and often with imm!noglob!lin %*g'1 antibodies. Treatment may be s!ccessf!l with azathioprine. &emphigoid is associated with s!bepidermal b!llae. --------------------->!estion: 43 of 193 Time taken: 09:13:55

89

5 33-year-old female attends her 1& concerned regarding a mole. "hich of the following characteristics of the lesion wo!ld raise s!spicion that it is a malignant melanoma$ %&lease select 1 option' :esion has irreg!lar edge )orrect)orrect :esion is 5 mm in diameter :esion is pigmented !niformly :esion is present on face :esion is smoothly raised The mnemonic of 5B),. regarding characteristics of a melanoma are as follows: 5 - 5symmetry - one half of the lesion does not match the other half B - Border irreg!larity ) - )olo!r variegation - pigmentation is not !niform , - ,iameter- a diameter 4 mm warrants investigation altho!gh changes in size are also important . - .vol!tion - evolving size or changes in characteristics s!ch as nod!les. ------------------------->!estion: 48 of 193 Time taken: 09:19:04 5 1?-year-old boy presents with scaly patches on his scalp. .2amination reveals well-circ!mscribed/ circ!lar areas of hair loss/ 7-5 cm in diameter with scaling and raised margins. There is no scarring. "hat is the most likely ca!se in this patient$ %&lease select 1 option' ,iscoid l!p!s erythemato!s *ncorrect*ncorrect answer selected 85

:ichen plan!s Corphoea ystemic l!p!s erythemato!s Tinea capitis This is the correct anwserThis is the correct answer This patient had non-scarring alopecia d!e to invasion of hairs by dermatophytes/ most commonly Trichophyton tons!rans. The most common ca!ses of nonscarring alopecia incl!de telogen effl!vi!m/ androgenetic alopecia/ alopecia areata/ tinea capitis// and tra!matic alopecia. :ess commonly/ non-scarring alopecia is associated with l!p!s erythematos!s and secondary syphilis. carring alopecia is more fre#!ently the res!lt of a primary c!taneo!s disorder s!ch as lichen plan!s/ follic!litis decalvans/ c!taneo!s l!p!s/ or linear scleroderma %morphea'. ----------------------->!estion: 30 of 193 Time taken: 09:19:15 5 77-year-old man cons!lts his 1& complaining of redness and itching on his face and hands. (e has reg!larly visited another 1& for similar complaints within the last two years and has been signed off on sick leave from employment as a b!ilder. (e is in receipt of benefits and reports that he is in the process of making an ins!rance claim for loss of earnings. (e says that there was one ointment that c!red the problem b!t he had not been able to find any proprietary medication that works. .2amination showed no skin lesions and no apparent rash.

8?

"hat is the most likely diagnosis$ %&lease select 1 option' )onversion disorder *ncorrect*ncorrect answer selected ,el!sional disorder Calingering This is the correct anwserThis is the correct answer C!ncha!sen syndrome +bsessive comp!lsive disorder *n somatoform disorders/ both illness prod!ction and motivation are !nconscio!s drives. *n malingering/ as s!ggested here/ the patient conscio!sly fakes or claims to have a disorder in order to attain a specific gain %for e2ample/ financial'. C!ncha!sen@s syndrome is manifested by a chronic history of m!ltiple hospital admissions and willingness to receive invasive proced!res. NNNNNNNNNNNNNNNNNN >!estion: 31 of 193 Time taken: 09:19:13 5 95-year-old teacher presents si2 weeks after he ret!rns from a hiking holiday in o!th 5merica with a shallow/ painless !lcer of the nose. "hat is the likely diagnosis$ %&lease select 1 option' 6!sobacteri!m !lcerans *ncorrect*ncorrect answer selected :eishmaniasis This is the correct anwserThis is the correct answer #!amo!s cell carcinoma Trichomoniasis Trypanosomiasis

84

The likely diagnosis/ given the history/ is c!taneo!s leishmaniasis. :esion pain and pr!ritis may be present in c!taneo!s leishmaniasis/ b!t are not typical. ,iagnosis is by histologic section with staining for amastigotes. :eishmania braziliensis is the likely pathogen which is spread by sandfly bites in endemic areas. 6!sobacteria ca!se the tropical !lcer/ an intensely painf!l/ shallow !lcer. ---------------------------->!estion: 37 of 193 Time taken: 09:19:3? 5 75-year-old female presents with concerns regarding the !nsightly appearance of her toe nails. They have a whitish discolo!ration e2tending !p the nail bed in a n!mber of the toes of both feet. They are entirely painless and she is otherwise well. "hat is the most appropriate treatment$ %&lease select 1 option' +ral fl!conazole *ncorrect*ncorrect answer selected +ral terbinafine This is the correct anwserThis is the correct answer Topical benzoic acid Topical fl!conazole Topical terbinafine This yo!ng woman has typical feat!res of f!ngal nail infection %onychomycosis' and the most appropriate treatment is oral antif!ngals as topical antif!ngals may be effective for one or two nails b!t not where there are a n!mber affected.

83

/u"t(e" .ea'in*: -oberts ,T/ Taylor ",/ Boyle GH British 5ssociation of ,ermatologists. 1!idelines for treatment of onychomycosis. Br J Dermatol 7003H193:907-910 ------------------------->!estion: 33 of 193 Time taken: 09:19:54 5 90-year-old female presents with a long history of e2cessive localised armpit sweating. he finds the problem embarrassing and has problems staining clothes. he has tried antiperspirants witho!t relief. "hich is the most appropriate treatment that yo! wo!ld offer this patient$ %&lease select 1 option' 5mitriptyline *ncorrect*ncorrect answer selected 52illary s!rgery Bot!lin!m to2in inAection This is the correct anwserThis is the correct answer &ropantheline Topical al!mini!m salts This woman has primary hyperhidrosis which can be #!ite psychologically disabling. *n this case the most appropriate treatment wo!ld be bot!lin!m to2in inAection to each a2illa. This treatment is licensed for !se and wo!ld be the preferred treatment before al!mini!m salts/ as antiperspirants have failed. imilarly/ antim!scarinics are associated with systemic side effects that may prove intolerable and their efficacy is really not proven.

88

&harmacological approaches sho!ld be tried before s!rgery. ------------------->!estion: 39 of 193 Time taken: 09:15:14 5 70-year-old female with a history of systemic l!p!s erythematos!s presents with symmetrical retic!lated/ violaceo!s patches/ which become more prominent in cold weather involving both lower limbs. "hich of the following is the likely diagnosis$ %&lease select 1 option' .rythema ab igne *ncorrect*ncorrect answer selected .rythema marginat!m .rythema nodos!m :ivedo retic!laris This is the correct anwserThis is the correct answer &yoderma gangrenos!m :ivedo retic!laris is d!e to dilation of capillary blood vessels and stagnation of blood within these vessels prod!cing a mottled discolo!ration of the skin. *t is described as being retic!lar %net-like' cyanotic c!taneo!s discolo!ration s!rro!nding pale central areas. *t occ!rs mostly on the legs/ arms and tr!nk and is more prono!nced in cold weather. Costly it is idiopathic/ or secondary to Calignancy 0asc!litis :. )ancer and )holesterol embolisation. ----------------->!estion: 35 of 193 Time taken: 09:15:11

100

5 47-year-old woman presents with a longstanding leg !lcer. "hich of the following minerals is most important in wo!nd healing$ %&lease select 1 option' )opper *ncorrect*ncorrect answer selected Cagnesi!m &otassi!m eleni!m Einc This is the correct anwserThis is the correct answer )ertain s!pplements are important in wo!nd healing partic!larly zinc/ vitamin ) and arginine. Einc is a component of many of the enzymes responsible for wo!nd healing. ------------------>!estion: 3? of 193 Time taken: 09:15:3? 5n 37-year-old lady had a history of a red facial rash and has s!ffered with veno!s eczema of the legs. he was treated for acne rosacea. +n e2amination/ she was noted to have bl!e-grey discolo!ration of both legs. "hat dr!g is most likely to have ca!sed this$ %&lease select 1 option' 5miodarone *ncorrect*ncorrect answer selected )iproflo2acin ,o2ycycline Cinocycline This is the correct anwserThis is the correct answer +2ytetracycline

101

This patient has developed skin pigmentation of her legs as a side effect of treatment of her acne rosacea. Tetracyclines are commonly !sed treatment for acne rosacea. :ong-term !se of minocycline in partic!lar has been associated with non-dose dependent bl!e-grey pigmentation of skin in the lower legs/ and m!cosal pigmentation. This is more common in the elderly. +n biopsy/ intracell!lar pigment is seen in the dermis and the s!bc!taneo!s tiss!e and stains positively for melanin and iron. *f not e2tensive/ hyperpigmentation may partially regress after minocycline is discontin!ed. *f it persists/ ale2andrite laser therapy can be effective. 5miodarone can also ca!se a bl!e-grey slate discolo!ration of the skin/ typically in s!n e2posed areas. Ko! wo!ld therefore e2pect the face to be affected/ and also amiodarone is not a treatment for acne rosacea which has been mentioned in this #!estion to lead yo! to the correct answer. (ypersensitivity to the s!n has been described with ciproflo2acin/ b!t discolo!ration as in this scenario is not recognised. +2ytetetracycline and do2ycycline can lead to photosensitivity/ b!t skin pigmentation seems to be specific to minocycline rather than a class effect of the tetracyclines. -eferences: (yperpigmentation associated with minocycline therapy. )hatterAee . )C5G %7004'H14?%3': -------------------->!estion: 34 of 193 Time taken: 09:15:35 5 77-year-old female presents with a si2 month history of increasing fatig!e and arthralgia of the wrists and ankles. Core recently/ she has also noted a symmetrical rash on her cheeks and some hair loss. "hat is the most likely diagnosis$

107

%&lease select 1 option' ,ermatomyositis *ncorrect*ncorrect answer selected (ypothyroidism &orphyria c!tanea tarda cleroderma ystemic l!p!s erythematos!s % :.' This is the correct anwserThis is the correct answer This woman has clinical feat!res consistent with systemic l!p!s erythematos!s. he gives a history of fatig!e which occ!rs commonly in :.. 5rthralgia and arthritis are the most common presenting manifestations of :. typically affecting the small Aoints of the hands/ wrists and knees. The symmetrical rash is the classical b!tterfly rash that occ!rs in a malar distrib!tion. 5lopecia is common and may be diff!se or patchy. *n dermatomyositis there is pro2imal/ symmetrical m!scle weakness that progresses over weeks to months. The typical lilac pap!lar rash occ!rs over the dors!m of the metacarpophalangeal %1ottron@s pap!les'/ eyelids/ elbows and knees. (ypothyroidism does not commonly res!lt in a symmetrical facial rash. The initial symptoms of scleroderma/ now termed systemic sclerosis tend to be non-specific and may consist of fatig!e/ weakness and m!sc!loskeletal complaints. -ayna!d@s phenomenon is an early symptom. kin changes incl!de telangiectasia/ hyper- and hypo-pigmentation. &orphyria c!tanea tarda is either primary or secondary !roporphyrinogen decarbo2ylase deficiency. The commonest symptoms are c!taneo!s fragility and blistering of s!n-e2posed skin. There may also be !rine discolo!ration. --------------->!estion: 33 of 193 103

Time taken: 09:1?:1? 5 31-year-old female presents with red scaly pla#!es on her cheeks/ forehead and sides of the neck. +n close inspection of the lesions there was pl!gging of some hair follicles with keratin and atrophy of the skin. "hat is the most likely diagnosis$ %&lease select 1 option' 5topic eczema *ncorrect*ncorrect answer selected ,iscoid l!p!s erythematos!s This is the correct anwserThis is the correct answer &olymorphic light er!ption &orphyria c!tanea tarda &soriasis This woman has discoid l!p!s erythematos!s. :esions are discrete pla#!es/ often erythemato!s/ covered by scales that e2tend into dilated hair follicles. These lesions most typically occ!r on the face/ scalp/ in the pinnae/ behind the ears and on the neck. They can e2ist in areas not e2posed to the s!n. The lesions can progress/ with active ind!rated erythema at the periphery. )entral atrophic scarring is characteristic. "itho!t treatment/ lesions can ca!se permanent scarring and alopecia. &atients with widespread disease are at increased risk of developing systemic l!p!s erythematos!s and sho!ld be closely followed !p. 6l!ocinonide cream/ hydrocortisone and acitretin are topical treatment options.

109

*n eczema dryness and lichenification are predominant feat!res. &soriasis commonly appears as inflamed lesions covered with a silvery white scale. &olymorphic light er!ption is characterised by rec!rrent/ abnormal/ delayed reactions to s!nlight/ ranging from erythemato!s pap!les/ pap!lovesicles/ and pla#!es to erythema m!ltiforme-like lesions on s!nlight-e2posed s!rfaces. &orphyria c!tanea tarda is either primary or secondary !roporphyrinogen decarbo2ylase deficiency. The commonest symptoms are c!taneo!s fragility and blistering of s!n-e2posed skin. There may also be !rine discolo!ration. -eferences: ,r!gs for discoid l!p!s erythematos!s. Gessop yst -ev. 7008 +ct 4H%9':),007859 ------------------->!estion: 38 of 193 Time taken: 09:1?:5? 5 77-year-old female is referred with symmetrical/ depigmented areas on the arms and legs b!t has been otherwise #!ite well. "hich one of the following diseases is most likely to be associated with her skin disease$ %&lease select 1 option' ,iabetes mellit!s *ncorrect*ncorrect answer selected (yperparathyroidism &ernicio!s anaemia This is the correct anwserThis is the correct answer cleroderma ystemic l!p!s erythematos!s % :.' 105 et al. )ochrane ,atabase

This lady has vitiligo/ which has a n!mber of disease associations. *t is the most common depigmenting disorder/ and affects 0.5I of the world pop!lation. (alf present before the age of 70/ and there is no difference in rate between se2es/ skin type or race. There seems to be a genetic basis to the disease/ and familial cl!stering is seen. The maAority of diseases associated with vitiligo are a!toimm!ne. Thyroid disease %partic!larly (ashimoto thyroiditis and 1raves disease' is one of the strongest associations/ and is often screened for in patients with vitiligo. (:5-B13 appears to be the link between thyroid disease and vitiligo. 8I of patients with pernicio!s anaemia have been shown to have vitiligo/ compared to 5.4I with diabetes mellit!s. +ther associations are 5ddison@s disease/ alopecia areta/ rhe!matoid arthritis/ inflammatory bowel disease and psoriasis. *n addition/ vitiligo may be a manifestation of polygland!lar a!toimm!ne syndrome/ which incl!des (K&+parathyroidism rather than hyperparathyroidism. :. can ca!se a post-inflammatory hypopigmentation/ b!t neither it or scleroderma have a recognised association with vitiligo. -eferences O 6!rther -eading: 0itiligo. Taieb 5 and &icardo C. =.GC %7008'H3?0:1?0-1?8 0itiligo and &ernicio!s 5nemia. .gli 6 and "alter -. =.GC %7009'H350:7?83 ---------------------------------------->!estion: 80 of 193 Time taken: 09:14:0? 5 14-year-old pregnant female attends antenatal clinic and is noted to have scattered small/ raised lesions on her tr!nk and a2illary freckles.

10?

he was not aware of any of her family members having these lesions. "hat is the likely mode of inheritance of this condition$ %&lease select 1 option' 5!tosomal dominant )orrect)orrect 5!tosomal recessive Trin!cleotide repeating P linked dominant P linked recessive This patient has ne!rofibromatosis with a2illary freckling and ne!rofibromas. This is !s!ally inherited as a!tosomal dominant/ altho!gh it may arise from a sporadic new m!tation of the =61 gene. --------------------------------------------->!estion: 81 of 193 Time taken: 09:14:13 5 77-year-old female ret!rns from a fortnight holiday in )ypr!s with a tan and n!mero!s scaly hypopigmented lesions on the neck and !pper tr!nk. "hat is the most likely diagnosis$ %&lease select 1 option' )hronic pla#!e psoriasis *ncorrect*ncorrect answer selected ,iscoid eczema &ityriasis rosea &ityriasis versicolor This is the correct anwserThis is the correct answer eborrhoeic dermatitis &ityriasis versicolor is ca!sed by a s!perficial f!ngal infection with &ityrospor!m ovale. 104

*t !s!ally presents as slightly scaly hypopigmented lesions. 1rowth is enco!raged by an increase in temperat!re and s!ntan oils/ and is most commonly seen after a s!n holiday. )hronic pla#!e psoriasis/ discoid eczema and seborrhoeic dermatitis have distinct appearance and distrib!tion. &ityriasis rosea !s!ally starts with a herald patch followed by small scaly lesions following the rib lines. ---------------------------------------------->!estion: 87 of 193 Time taken: 09:14:53 5 57-year-old female presents with blistering of the hands and arms which deteriorates d!ring the s!mmer. he is otherwise well and drinks appro2imately 70 !nits of alcohol weekly. .2amination of her skin revealed erosions and scarring on the backs of her hands and forearms/ and some mild hirs!tism. "hich one of the following is the most likely diagnosis$ %&lease select 1 option' 5c!te intermittent porphyria *ncorrect*ncorrect answer selected .rythropoietic protoporphyria &emphigoid &orphyria c!tanea tarda %&)T' This is the correct anwserThis is the correct answer !bac!te l!p!s erythemato!s %:.' The history of photosensitive er!ption/ hypertrichosis and milia formation %keratin-containing cysts' are characteristic of &)T. 103

5c!te intermittent porphyria !s!ally ca!ses intermittent ne!rological and abdominal problems. !bac!te :. can ca!se erythema and scarring/ b!t in a patient with increased alcohol intake and hypertrichosis the most likely diagnosis is &)T. .rythropoietic protoporphyria patients get pain and erythema on e2pos!re to s!nlight. ------------------------------------->!estion: 83 of 193 Time taken: 09:14:55 5 33-year-old female is admitted with erythema m!ltiforme %.C' and erythemato!s lesions of the mo!th and eyes. "hich one of the following dr!gs may acco!nt for her presentation$ %&lease select 1 option' ,iazepam *ncorrect*ncorrect answer selected 6l!o2etine Cebeverine +ral contraceptive !lfasalazine This is the correct anwserThis is the correct answer 5ny dr!g or infection can trigger .C b!t s!lfasalazine and s!lfa-gro!p dr!gs are well reported as ca!ses of .C and tevens-Gohnson syndrome. ----------------------------------------------->!estion: 89 of 193 Time taken: 09:14:55

108

5 75-year-old female presents with red cr!sted lesions aro!nd the mo!th and finger p!lps/ three months after having had small bowel resection for )rohn@s disease. "hat is the most likely ca!se of her skin condition$ %&lease select 1 option' =icotinamide deficiency *ncorrect*ncorrect answer selected &yrido2ine deficiency Thiamine deficiency 0itamin B17 deficiency Einc deficiency This is the correct anwserThis is the correct answer Einc deficiency can lead to acrodermatitis which presents with perioral dermatitis/ acral involvement and sometimes alopecia. The feat!res of this patient who had bowel resection s!ggest zinc deficiency. ------------------------------------->!estion: 85 of 193 Time taken: 09:14:55 5 1?-year-old girl is seen in clinic as she is concerned d!e to areas of hair loss on the scalp. &ast medical history incl!des atopic eczema and she has a n!mber of depigmented areas on her hands. "hat is the most likely diagnosis$ %&lease select 1 option' 5lopecia areata )orrect)orrect (ypothyroidism eborrhoeic dermatitis 110

ystemic l!p!s erythematos!s Trichotillomania This girl has a combination of vitiligo and alopecia areata which can co-e2ist and have similar a!toimm!ne aetiology. ,iscrete areas of hair loss and normal te2t!re on the scalp are highly s!ggestive of alopecia areata. ---------------------------------->!estion: 8? of 193 Time taken: 09:79:15 5 40-year-old female presents with a fo!r month history of a dry/ pr!ritic rash affecting the !pper back and shins. "hat is the most appropriate initial management of this patient$ %&lease select 1 option' 5voidance of contact irritants *ncorrect*ncorrect answer selected .mollients This is the correct anwserThis is the correct answer kin biopsy Take a detailed history to ascertain contact allergen Topical corticosteroids This lady is likely to have asteatotic eczema which is a common problem and will improve A!st with plain emollients. Perotic skin is commoner in the elderly pop!lation especially in the winter months d!e to the dry heat from central heating. 5ll the other s!ggestions may be appropriate in a patient resistant to first line treatment b!t the first line is to try emollients. -----------------------------------------------111

>!estion: 84 of 193 Time taken: 09:79:18 5 14-year-old girl presents with a two week history of !rticaria. +ver the last co!ple of days she has been aware of new lesions occ!rring on a daily basis. "hich one of the following statements is most likely to be correct$ %&lease select 1 option' he is likely to have an associated asthma *ncorrect*ncorrect answer selected he is likely to have taken penicillin recently he is !nlikely to have any identifiable trigger factor This is the correct anwserThis is the correct answer The lesions will be present for at least 79 ho!rs There is likely to be a n!t allergy The commonest form of !rticaria is idiopathic and there is no identifiable trigger. &ean!t allergy and penicillin may ca!se !rticarial rashes b!t there is !s!ally an associated specific history of contact with the allergen. Frticarial lesions last less than 79 ho!rs !s!ally. ------------------------------------------------>!estion: 83 of 193 Time taken: 09:79:73

117

5 3?-year-old female presents with raised erythemato!s tender lesions on both legs which have developed since she had a throat infection two weeks ago. "hich one of the following investigations is most likely to establish the diagnosis$ %&lease select 1 option' 5nti-streptolysin-+ titre %5 +T' )orrect)orrect )hest 2 ray Canto!2 test Throat swab c!lt!red for bacteria Throat swab c!lt!red for vir!ses This lady presents with tender lesions two weeks after a sore throat. This is most likely to be post-streptococcal erythema nodos!m and 5 +T is most likely to confirm this. --------------------------------->!estion: 88 of 193 Time taken: 09:75:03 5 91-year-old female presents with a si2 month history of a pr!ritic vesic!lar-pap!lar rash on the elbows/ knees and b!ttocks associated with n!mero!s blistering er!ptions and e2coriations. (er 1& has prescribed topical steroid therapy b!t this has not helped. "hat is the most likely diagnosis$ %&lease select 1 option' 5topic eczema %dermatitis' *ncorrect*ncorrect answer selected

113

,ermatitis herpetiformis %,(' This is the correct anwserThis is the correct answer :ichen plan!s &soriasis cabies This patient presents with pr!ritic vesicles on her elbows/ knees and b!ttocks which have not responded to topical steroids. This is the classical presentation of ,(. 5topic dermatitis !s!ally is fle2!ral and responds to topical steroids. (enoch- chQnlein p!rp!ra is a form of vasc!litis. *n scabies there are b!rrows and in psoriasis the rash consists of pla#!es with silvery scales. ------------------------>!estion: 100 of 193 Time taken: 09:75:03 5 14-year-old boy is diagnosed with scabies. "hich of the following statements regarding scabies is correct$ %&lease select 1 option' *s best treated by salicylate em!lsion *ncorrect*ncorrect answer selected *t can be spread by a droplet infection *t ca!ses itchiness in the skin even where there is no obvio!s lesion to be seen This is the correct anwserThis is the correct answer *t is ca!sed by taphylococc!s a!re!s Typically affects the face

119

cabies is an infestation of the skin with the microscopic mite arcoptes scabiei. *nfestation is common/ fo!nd worldwide/ and affects people of all races and social classes. cabies spreads rapidly !nder crowded conditions where there is fre#!ent skin-to-skin contact between people/ s!ch as in hospitals/ instit!tions/ childcare facilities/ and n!rsing homes. cabies can spread by direct/ prolonged/ skin-to-skin contact/ with a person already infested with scabies. )ontact m!st be prolonged %a #!ick handshake or h!g will !s!ally not spread infestation'. *nfestation is easily spread to se2!al partners and ho!sehold members. *nfestation may also occ!r by sharing clothing/ towels/ and bedding. cabies is characterised by pap!lar-like irritations/ b!rrows or rash of the skin/ especially the webbing between the fingersH the skin folds on the wrist/ elbow/ or kneeH the penis/ the breast/ or sho!lder blades. 5 n!mber of treatments are available for the treatment of scabies/ incl!ding permethrin ointment/ benzyl benzoate/ and oral ivermectin for resistant cases. 5ntihistamines and calamine lotion may be !sed to alleviate itching. -------------------------------------->!estion: 101 of 193 Time taken: 09:75:08 5 15-year-old boy was treated with permethrin cream for scabies infestation. +n follow-!p three weeks later he was fo!nd to have contin!ing infestation. "hat is the most likely reason for this$ %&lease select 1 option' 115

6acial skin was not treated *ncorrect*ncorrect answer selected =on-disposal of !nderwear +ther ho!sehold members were not treated This is the correct anwserThis is the correct answer The organism is resistant to permethrin The treatment was not repeated as prescribed cabies is an intensely pr!ritic and highly contagio!s infestation of the skin ac#!ired thro!gh close personal contact. 5 delayed type *0 hypersensitivity reaction to the mites/ their eggs/ or e2creta occ!rs appro2imately 30 days after infestation and is responsible for the intense pr!rit!s that is the hallmark of the disease. 5ll ho!sehold members and close personal contacts sho!ld be treated whether or not they are symptomatic and patients sho!ld be re-e2amined two weeks after treatment to eval!ate effectiveness. Treatment fail!res are !ncommon. -ec!rrence of the er!ption !s!ally means re-infection has occ!rred. ------------------------------------->!estion: 107 of 193 Time taken: 09:75:53 *n the treatment of psoriasis/ which of the following statements is correct$ %&lease select 1 option' .tanercept is not effective in psoriatic arthritis *ncorrect*ncorrect answer selected *nfli2imab is associated with t!berc!losis This is the correct anwserThis is the correct answer &F05 is ineffective -etinoids are the most !sef!l monotherapy in psoriatic patients 11?

0itamin , analog!es are associated with c!taneo!s atrophy &soralen and !ltraviolet light %&F05' is an effective treatment for psoriasis b!t has been related to increased risk of s#!amo!s cell carcinoma/ and possibly malignant melanoma. -etinoids are most effective in combination therapy especially with !ltraviolet B %F0B' phototherapy and &F05. &art of the attraction of vitamin , analog!es over steroids is that they do not ca!se c!taneo!s atrophy whereas steroids do. The recombinant t!mo!r necrosis factor and receptor f!sion protein/ etanercept/ has demonstrated considerable beneficial effects on psoriatic arthropathy in placebo controlled/ do!ble blind st!dies. *nfli2imab is associated with t!berc!losis by reactivation of latent disease. Th!s it is advised that patients who are to be treated with infli2imab are t!berc!lin tested and if re#!ired receive chest radiography. -------------------------------------->!estion: 103 of 193 Time taken: 09:75:59 "hich of the following statements regarding psoriasis is most tr!e$ %&lease select 1 option' )iclosporin is ineffective in the treatment of psoriasis *ncorrect*ncorrect answer selected ,iagnosis re#!ires histological confirmation 1!ttate psoriasis often arises after staphylococcal infection T cells play a prominent role in the pathogenesis of psoriasis This is the correct anwserThis is the correct answer 114

Twin st!dies have identified no genetic basis for psoriasis ,iagnosis of psoriasis is based on clinical observation of sharply demarcated/ erythemato!s/ scaling pla#!es/ and rarely re#!ires biopsy. treptococcal infection is associated with precipitation and rec!rrence of g!ttate psoriasis. )iclosporin is a maAor inhibitor of T cell activation/ and/ given that T cells are central to the pathogenesis of psoriasis/ is very effective treatment in psoriasis. 1enetic st!dies have led most e2perts to believe that psoriasis is the res!lt of m!ltiple genetic factors interacting with environmental stim!li. 1enetic factors also seem to contrib!te to the clinical manifestations of the disease/ for e2ample/ age of onset and severity of disease. -------------------------------------------->!estion: 109 of 193 Time taken: 09:7?:0? "hich of the following statements regarding psoriasis is correct$ %&lease select 1 option' 1I of patients have associated psoriatic arthropathy *ncorrect*ncorrect answer selected 1!ttate psoriasis is the most common form of the disease &soriasis is more common at lower geographical altit!des &soriatic arthropathy precedes c!taneo!s lesions in ro!ghly 70I of cases This is the correct anwserThis is the correct answer The prevalence in the Fnited Dingdom is 10I The prevalence of psoriasis is reported as between 0.5 and 9.?I.

113

6or reasons which may be e2plained by the filtering of !ltraviolet B %F0B' light/ psoriasis is more common at higher altit!des. The commonest form of psoriasis is pla#!e psoriasis/ making !p appro2imately 30I of cases %g!ttate -10I/ erythrodermic - 3I/ p!st!lar 3I'. t!dies report: M 5 5-97I prevalence of psoriatic arthropathy in patients with c!taneo!s psoriasis and M 5rthropathy precedes c!taneo!s lesions in 70I M )!taneo!s lesions precede Aoint disease in ?0-40I and M They occ!r sim!ltaneo!sly in 10-70I. --------------------------------------Question: 105 of 148 Time taken: 09:7?:54 5 ?8-year-old woman presented with an !lcer over the left ankle/ which had developed over the previo!s nine months. he had a history of right deep vein thrombosis %,0T' five years previo!sly. +n e2amination she had a s!perficial slo!gh-based !lcer/ ? cm in diameter/ over the medial malleol!s with no evidence of cell!litis. "hich one of the following is the most appropriate ne2t investigation$ %&lease select 1 option' 5nkle-brachial press!re inde2 )orrect Bacteriological swab of the !lcer Bilateral lower limb arteriogram -ight leg venogram 0eno!s d!ple2 !ltraso!nd scan *t has been reported that veno!s !lcerations are the most common type of !lcer affecting the lower e2tremities. The probable !nderlying ca!se of veno!s congestion/ which may promote !lceration/ is veno!s ins!fficiency.

118

The treatment of veno!s !lceration is control of oedema/ treating any infection/ and compression. (owever/ compressive dressings or devices sho!ld not be applied if the arterial circ!lation is impaired. *t is th!s important to identify any arterial disease/ and ankle-brachial press!re inde2 is a simple way of doing this. +ne may then progress to lower limb arteriogram if indicated. There is no clinical sign of infection/ and altho!gh a bacterial swab wo!ld help to r!le o!t pathogens within the !lcer/ arterial ins!fficiency is the more important iss!e. *f there is a clinical s!spicion of ,0T then d!ple2 %or rarely a venogram' is indicated to decide on the indication for warfarin. -------------------->!estion: 10? of 193 Time taken: 09:74:03 5 35-year-old woman presents with a facial rash which had been present for one year. +n e2amination she had erythemato!s/ scaly/ ind!rated pla#!es on both cheeks with areas of scarring alopecia. (yperkeratosis over dilated hair follicles was also seen. "hat is the diagnosis$ %&lease select 1 option' 5cne rosacea *ncorrect*ncorrect answer selected ,iscoid l!p!s erythematos!s This is the correct anwserThis is the correct answer *mpetigo :!p!s pernio &soriasis

170

The patient has discoid l!p!s as s!ggested by the ind!rated pla#!es on cheeks/ the scarring alopecia and hyperkeratosis over the hair follicles. ------------------->!estion: 104 of 193 Time taken: 09:74:1? 5 79-year-old female attends clinic complaining of n!mero!s depigmented areas on the arms and legs. "hich of the following diseases is most likely to accompany this skin condition$ %&lease select 1 option' 5ddison@s disease *ncorrect*ncorrect answer selected (ypoparathyroidism &ernicio!s anaemia This is the correct anwserThis is the correct answer ystemic l!p!s erythematos!s T!bero!s sclerosis The s!ggested diagnosis is vitiligo which is associated with n!mero!s a!toimm!ne conditions incl!ding/ in order of fre#!ency: 1. 5!toimm!ne hypothyroidism 7. &ernicio!s anaemia 3. 5lopecia areata 9. 5ddison@s disease. *t is associated with both type 1 and 7 a!toimm!ne polyendocrine syndromes b!t these are m!ch rarer than the former diagnoses. ------------------->!estion: 103 of 193 Time taken: 09:74:19 171

5n 13-year-old woman attends antenatal clinic 17 weeks into her pregnancy where the doctor incidentally notes n!mero!s small l!mps over her tr!nk and freckles in her a2illae. he reported that none of her relatives had any similar feat!res. "hat is the most likely diagnosis$ %&lease select 1 option' 5canthosis nigricans *ncorrect*ncorrect answer selected ,ysplastic naev!s syndrome Castocytosis =e!rofibromatosis This is the correct anwserThis is the correct answer T!bero!s sclerosis The patient is likely to have ne!rofibromatosis %=61'. To be given the diagnosis of =61/ an individ!al m!st have at least two of the following feat!res: M i2 or more cafL-a!-lait spots M Two or more ne!rofibromas or a ple2iform ne!rofibroma M 52illary freckling %can also be present in the groins' M +ptic glioma M :isch nod!les %visible within the iris' M 5 family history 5canthosis nigricans is velvety hyperpigmentation !s!ally present within the a2illae/ and associated with obesity or endocrinologies. ,ysplastic naev!s syndrome is a familial c!taneo!s condition which is characterised by atypical naevi and melanomas.

177

Castocytosis describes the proliferation and acc!m!lation of mast cells within the organs. *n the skin it can present with !rticaria/ mac!lopap!lar lesions and diff!se infiltration. T!bero!s sclerosis is characterised by the formation of hamartomas in a n!mber of different organisms/ incl!ding the brain/ skin and kidney. ------------------>!estion: 108 of 193 Time taken: 09:74:08 5 55-year-old woman presents with a non-pr!ritic rash that had developed over the last two months. .2amination revealed several/ circ!lar/ erythemato!s/ raised/ smooth-s!rfaced lesions of variable size from 1-5 cm in diameter on the elbows/ e2tensor aspects of the forearms and kn!ckles. "hat is the most likely diagnosis$ %&lease select 1 option' .czema *ncorrect*ncorrect answer selected 1ran!loma ann!lare This is the correct anwserThis is the correct answer &soriasis Tinea corporis Frticaria The history of non-itchy/ circ!lar/ raised/ smooth-s!rfaced lesions on the elbows/ e2tensor aspects of the forearms and kn!ckles and the raised borders are s!ggestive of gran!loma ann!lare. ,iscoid eczema tends to be scaly and pr!ritic in nat!re. &soriasis typically has a silvery scale and can be pr!ritic. Frticaria lasts a few ho!rs and is pr!ritic. 173

Tinea corporis is a f!ngal infection and is typically scaly and pr!ritic in nat!re. The most likely answer is therefore gran!loma ann!lare. This can be associated with diabetes. -------------------->!estion: 110 of 193 Time taken: 09:74:08 5 40-year-old woman complained of a rash that had developed over a month. he had otherwise been fit and well. +n e2amination/ there were n!mero!s tense/ fl!id filled blisters over the tr!nk and limbs/ b!t no m!cosal involvement was evident. "hat is the most likely diagnosis$ %&lease select 1 option' ,ermatitis herpetiformis *ncorrect*ncorrect answer selected .rythema m!ltiforme (erpes simple2 &emphigoid This is the correct anwserThis is the correct answer &emphig!s v!lgaris The patient presents with tense blisters on her arms/ tr!nk and legs. he is otherwise well and there is no m!cosal involvement. This is typical of b!llo!s pemphigoid. ,ermatitis herpetiformis presents with itchy e2coriated areas in the elbows knees and b!ttocks. .rythema m!ltiforme presents with characteristic target lesions.

179

(erpes simple2 is vesic!lar and in generalised cases the patient is likely to be !nwell. &emphig!s presents with s!perficial erosions and !s!ally there is m!cosal involvement. ------------------------------------>!estion: 111 of 193 Time taken: 09:74:08 5 ?0-year-old woman presents with raised/ erythemato!s lesions on the limbs and blistering in the mo!th and eyes. he had been taking a n!mber of dr!gs prescribed by her 1&. "hich may be responsible for her presentation$ %&lease select 1 option' =ifedipine *ncorrect*ncorrect answer selected &aracetamol &aro2etine &rednisolone !lfasalazine This is the correct anwserThis is the correct answer This is a typical case of tevens-Gohnson syndrome. tevens-Gohnson syndrome % G ' is an imm!ne-comple2-mediated hypersensitivity comple2 that is a severe e2pression of erythema m!ltiforme. *t is now known also as erythema m!ltiforme maAor. G typically involves the skin and the m!co!s membranes. "hile minor presentations may occ!r/ significant involvement of: M +ral M =asal 175

M .ye M 0aginal M Frethral M 1astrointestinal %1*' M :ower respiratory tract m!co!s membranes may develop in the co!rse of the illness. 1* and respiratory involvement may progress to necrosis. G is a serio!s systemic disorder with the potential for severe morbidity and even death. The dr!gs most closely associated with ca!sing tevens-Gohnson syndrome are: M 5ntibacterials M !lfonamides M 5nticonv!lsants %o2icam' M =on-steroidal anti-inflammatory agents %piro2icam and teno2icam' M )hlormezanone M 5llop!rinol. ----------------------------->!estion: 117 of 193 Time taken: 09:74:34 5 ?5-year-old female who has a history of long-standing psoriasis and heavy alcohol intake/ presents with a severe e2acerbation of psoriasis. he was admitted and received topical therapy and over the month of her admission/ her gamma-1T concentration had fallen from 900 F<: to 150 F<: %9-35'. i2 weeks after discharge she was seen in o!tpatients where her psoriasis remained !nder control/ b!t she complained of generalised hair loss. 17?

"hat is the most likely ca!se for her hair loss$ %&lease select 1 option' 5lopecia areata *ncorrect*ncorrect answer selected *ron deficiency Telogen effl!vi!m This is the correct anwserThis is the correct answer Thiamine deficiency Trichotillomania *n a normal healthy person@s scalp abo!t 35I of the hair follicles are actively growing hair and 15I are resting. *f there is some shock to the system/ as many as 40I of the scalp hairs can be precipitated into a resting state/ th!s reversing the !s!al ratio. Typical precipitants incl!de illnesses/ operations/ accidents and childbirth. The resting scalp hairs/ now in the form of cl!b hairs/ remain firmly attached to the hair follicles at first. *t is only abo!t two months after the shock that the new hairs coming !p thro!gh the scalp p!sh o!t the @dead@ cl!b hairs and increased hair fall is noticed. Th!s/ parado2ically/ with this type of hair loss/ hair fall is a sign of hair regrowth. 5s the new hair first comes !p thro!gh the scalp and p!shes o!t the dead hair a fine fringe of new hair is often evident along the forehead hairline. 5t first the fall of cl!b hairs is prof!se and a general thinning of the scalp hair may become evident b!t after several months a peak is reached and hair fall begins to lessen/ grad!ally tapering back to normal over si2 to nine months. 5s the hair fall tapers off the scalp thickens back !p to normal/ b!t recovery may be incomplete in some cases. --------------------------------------->!estion: 113 of 193 174

Time taken: 00:08 5 70-year-old male presents with e2tensive/ coalescing/ hypopigmented/ slightly scaly lesions on his back and chest. The rash had been present for two years and had grad!ally become more e2tensive. (e had otherwise been in good health. The lesions were not symptomatic b!t he was concerned abo!t their appearance. "hat is the most appropriate treatment for his condition$ %&lease select 1 option' 5ciclovir cream *ncorrect*ncorrect answer selected Detoconazole cream This is the correct anwserThis is the correct answer =ystatin cream +ral itraconazole +ral terbinafine The patient presents with an asymptomatic er!ption on his tr!nk. The lesions are scaly/ hypopigmented and are not associated with any systemic disease. This is characteristic of pityriasis versicolor/ which is ca!sed by the !nicell!lar yeast &ityrospor!m ovale and &ityrospor!m orbic!lare. The yeast is lipophilic and is enco!raged by an increase in environmental temperat!re/ th!s many patients notice that the condition begins after a s!mmer vacation. *t is a disorder of the healthy/ b!t the imm!nocompromised are at risk. The condition is asymptomatic and appears pale in comparison to the normal skin. The f!ng!s affects the melanocytes hence the hypo-pigmentation. The treatment options incl!de topical imidazole creams/ seleni!m s!lphide shampoo and/ if not responding to topical treatment/ oral itraconazole 700 mg once a day for seven days. 173

*n this patient the topical treatment sho!ld be tried first. ---------------------------------------Question: 114 of 148 Time taken: 00:34 5 ?3-year-old man presents with rec!rrent grad!ally increasing shortness of breath over the last few weeks. (is chest 2 ray is shown:

+n e2amination his hands showed the following:

178

"hat is the diagnosis$ %&lease select 1 option' )hronic m!coc!taneo!s candidiasis *ncorrect*ncorrect answer selected *ron deficiency +chronosis &olychondritis Kellow nail syndrome This is the correct anwserThis is the correct answer This is yellow nail syndrome where the nails are yellow/ thickened/ c!rved/ stop growing and may become detached from the nail bed. 5ssociated findings incl!de lymphoedema/ bronchiectasis and ple!ral eff!sions. ------------------------------------------>!estion: 115 of 193 Time taken: 00:9?

130

5 ?3-year-old woman presents with a two month history of a widespread pr!ritic rash. .2amination reveals widespread erythema with several small blisters containing straw-colo!red fl!id and one or two larger serosang!ineo!s blisters. "hat is the most likely diagnosis$ %&lease select 1 option' B!llo!s impetigo *ncorrect*ncorrect answer selected B!llo!s pemphigoid This is the correct anwserThis is the correct answer *nsect bite cabies Frticarial vasc!litis &emphigoid/ erythema m!ltiforme and herpes are the commonest ca!ses of a blistering rash. The history above is a classic description of b!llo!s pemphigoid %B&'. *mm!noglob!lin %*g'1 a!toantibodies bind to the skin basement membrane in patients with B&. The binding of antibodies at the basement membrane activates complement and inflammatory mediators. 5ctivation of the complement system is tho!ght to play a critical role in attracting inflammatory cells to the basement membrane. These inflammatory cells are post!lated to release proteases/ which degrade hemidesmosomal proteins and lead to blister formation. .osinophils are characteristically present in blisters as demonstrated by histopathologic analysis/ altho!gh their presence is not an absol!te diagnostic criterion. -----------------------------------------------131

>!estion: 11? of 193 Time taken: 00:5? 5 7?-year-old male epileptic is admitted with temperat!re and rash. +ver the last one week a rash has developed and he has become increasingly ill. -ecently he has had some problems with epileptic control and has commenced carbamazepine with valproate. .2amination reveals an !nwell patient with a temperat!re of 38R)/ a diff!se erythemato!s/ painf!l rash with evidence of some lateral sliding of these erythemato!s areas on palpation. There is also blistering and inflammation of the oral cavity. "hat is the likely diagnosis$ %&lease select 1 option' .rythema elevat!m di!tin!m *ncorrect*ncorrect answer selected .2foliative dermatitis &!st!lar psoriasis To2ic epidermal necrolysis This is the correct anwserThis is the correct answer To2ic shock syndrome This patient@s presentation and clinical description s!ggests a diagnosis of to2ic epidermal necrolysis %T.=' d!e to carbamazepine therapy. T.= is a severe m!coc!taneo!s e2foliative disease with an !ncertain pathogenesis and a high mortality rate. *t is diffic!lt to say whether it is another variant of tevens-Gohnson syndrome and treatment of both are similar. 137

*t is often idiopathic b!t may be associated with: M 0iral infections M :e!kaemia M :ymphoma and M ,r!gs %in partic!lar s!lphonamides and anticonv!lsants'. The s!ggested association with carbamazipine in this case makes to2ic shock syndrome d!e to taph. a!re!s remote/ which like p!st!lar psoriasis wo!ld not be e2pected to affect the m!co!s membranes. -------------------------------------->!estion: 114 of 193 Time taken: 01:79 5n 13-year-old woman presents with red/ tender l!mps on her shins and arthralgia. )hest 2 ray shows bilateral hilar lymphadenopathy and clear l!ng fields. 5 clinical diagnosis of sarcoidosis is made. "hich one of the following is the most appropriate management plan$ %&lease select 1 option' 79 ho!r !rinary calci!m meas!rement *ncorrect*ncorrect answer selected 6ollow !p appointment with chest 2 ray in three months This is the correct anwserThis is the correct answer Cediastinoscopy and lymph node biopsy kin biopsy Thoracic )T scan This is also known as :Qfgren@s syndrome %a benign form of sarcoidosis'. 133

The presentation of erythema nodos!m-arthropathy-bilateral hilar lymphadenopathy syndrome is so characteristic that histological diagnosis is not necessary. The prognosis is e2cellent with less than 10I having persistent disease. "ith less characteristic presentations/ positive biopsies are needed. Cediastinoscopy is the method of choice for anterior mediastinal nodes. ---------------------------------Question: 118 of 148 Time taken: 01:33 5 yo!ng woman has acne and is taking oral medication. he develops polyarthritis and has raised liver enzyme tests. *nvestigations show: 0+T 85 F<l %1-31' 0LT 140 F<l %5-35' %i&i"ubin 1? Smol<l %1-77' trongly positive at 1<70 0ntinu)&ea" antibo'ies =egative at 1<?90 "hich of the following dr!gs is she most likely to have been prescribed$ %&lease select 1 option' .rythromycin *ncorrect*ncorrect answer selected *sotretinoin Cinocycline This is the correct anwserThis is the correct answer +2ytetracycline Trimethoprim 5ll other dr!gs listed above can be !sed in the treatment of acne. 5ll of these can ca!se hepatoto2icity/ and therefore raised alanine aminotransferase %5:T' and aspartate aminotransferase %5 T'.

139

Cinocycline is the only dr!g listed which can acco!nt for the polyarthritis and antin!clear antibody %5=5'/ d!e to its ability to ca!se dr!g-ind!ced l!p!s erythematos!s. )lassically/ dr!g-ind!ced l!p!s erythematos!s is characterised by M ystemic disease with a lower incidence of nephritis M :ack of c!taneo!s involvement and M The presence of antihistone antibodies1. The most commonly associated dr!gs have historically been procainamide and hydralazine 7/ altho!gh their !se is now decreasing. Cedications associated more recently incl!de the anti-T=6 alpha agents/ statins and minocycline. Cinocycline is !n!s!al in that it seems to be associated with the development of long term imm!nological memory/ and therefore e2acerbation of symptoms within 17-79 ho!rs of rechallenge7. Cinocycline has been well doc!mented as a ca!se of dr!g-ind!ced systemic l!p!s erythematos!s % :.'. )haracteristically/ the erythrocyte sedimentation rate %. -' and ) reactive protein %)-&' are both markedly elevated/ the 5=5 is strongly positive and there is a hypergammaglob!linaemia. 5nti-ds,=5 antibodies are !s!ally negativeH antihistone antibodies are positive in 85I of dr!g-ind!ced l!p!s %b!t also 50-30I of idiopathic :.3'. 5 strongly positive 5=5 is a risk factor for developing dr!g-ind!ced l!p!s/ b!t a negative 5=5 wo!ld not e2cl!de the diagnosis7. ,r!g-ind!ced l!p!s is defined as a l!p!s-like syndrome temporally related to contin!o!s dr!g e2pos!re which resolves after discontin!ation of the offending dr!g9.

135

There are several feat!res which disting!ish dr!g-ind!ced l!p!s from idiopathic :.: M Cales and females are e#!ally affected in dr!g-ind!ced l!p!s/ whereas idiopathic :. affects females nine times more fre#!ently3. M )a!casians are affected by dr!g-ind!ced l!p!s more commonly than 5fro-)aribbeans/ whereas the inverse is tr!e of idiopathic :.. M *n addition/ the age of onset is typically older in dr!g-ind!ced l!p!s/ b!t this depends on the age at dr!g e2pos!re. M 6ever/ arthralgia/ serositis and 5=5 occ!r at least as fre#!ently in dr!gind!ced l!p!s as idiopathic :.. M (aematological/ renal and central nervo!s system %)= ' involvement/ and do!ble-stranded ,=5 a!toantibodies are rare3. The pathogenesis of dr!g-ind!ced l!p!s is !nclear. 6actors that infl!ence dr!g metabolism/ s!ch as acetylator stat!s/ have been implicated. *n addition/ l!p!s-ind!cing dr!gs have been shown to generate a variety of cytoto2ic prod!cts on e2pos!re to C&+ released from activated ne!trophils7. The time taken for symptoms to resolve after stopping minocycline is highly variable/ from a few days to two years3. Typically/ no f!rther treatment is re#!ired b!t there are sit!ations where corticosteroids or disease modifying antirhe!matic dr!gs %,C5-,s' are re#!ired to aid resol!tion3. -eferences: 1. ,r!g-*nd!ced/ -o< 5-&ositive )!taneo!s :!p!s .rythematos!s. rivastava C et al. 5rch ,ermatol 7003H138:95-98 7. Cinocycline-ind!ced l!p!s: clinical feat!res and response to rechallenge. :awson TC et al. -he!matology 7001H90%3':378-335 3. Cinocycline-ind!ced l!p!s: a case series. &orter , and (arrison 5. Go!rnal of the =ew Eealand Cedical 5ssociation 7003 5prilH11?%1141' 9. ,r!g-ind!ced l!p!s erythematos!s. 0edove ), et al. 5rch ,ermatol -es 7008 GanH301%1':88-105 13?

---------------------------------->!estion: 118 of 193 Time taken: 07:01 5 53-year-old man has a history of obesity/ gastro-oesophageal refl!2 disease/ low back pain and *(,. (e presents with large/ itchy weals over the tr!nk and limbs and a sensation of tightness in the throat. "hich one of the following dr!gs is the most likely to have triggered this skin er!ption$ %&lease select 1 option' 5spirin )orrect)orrect 1T= %nitrate' spray +meprazole &aracetamol imvastatin *n hypersensitive patients/ aspirin can ca!se: M 5ngioedema M Bronchospasm M Frticaria %skin rashes'. --------------------------------->!estion: 170 of 193 Time taken: 07:55 5 33-year-old female presents with red target lesions confined to the hands and is diagnosed with erythema m!ltiforme.

134

"hich of the following co!ld be the ca!se$ %&lease select 1 option' )ytomegalovir!s infection *ncorrect*ncorrect answer selected 1ro!p B streptococci :angerhan@s cells histiocytosis &enicillin 0 This is the correct anwserThis is the correct answer Freaplasma !realytic!m &otential ca!ses of erythema m!ltiforme incl!de: 1. *nfections M 0ir!ses: herpes simple2 1 and 7/ hepatitis B/ .pstein-Barr vir!s %.B0'/ enterovir!ses M mall agents: Cycoplasma pne!moniae M Bacteria: 1ro!p 5 treptococc!s/ eosina M +ther: Cycobacteri!m t!berc!losis/ histoplasma/ coccidioides. 7. =eoplasia M :e!kaemia M :ymphoma. 3. 5ntibiotics M &enicillins/ s!lphonamides/ isoniazid/ tetracycline. 9. 5nticonv!lsants M &henytoin/ phenobarbitone/ carbamazepine. 5. +ther

133

M 5spirin M -adiation therapy M .toposide M = 5*,s M !nlight M &regnancy. ------------------------------------------->!estion: 171 of 193 Time taken: 03:09 5 50-year-old man presented in the s!mmer complaining of itching and blistering of his hands and forehead. +n e2amination there were small areas of e2coriation on the backs of his hands. "hat is the most likely diagnosis$ %&lease select 1 option' ,ermatitis herpetiformis *ncorrect*ncorrect answer selected :!p!s erythematos!s &emphigoid &emphig!s &orphyria c!tanea tarda %&)T' This is the correct anwserThis is the correct answer The distrib!tion of the lesions s!ggests a photosensitive element. Both l!p!s erythematos!s and &)T are associated with a photosensitive element/ however this is more typical of &)T. &)T ca!ses blistering of the hands and the forehead which !s!ally heal with small scar and milia formation. 138

*t is also associated with an e2cessive alcohol intake. ---------------------------------------------------->!estion: 177 of 193 Time taken: 03:04 5 previo!sly fit 30-year-old female presents with a fo!r day history of intractable pr!rit!s and !rticaria. "hat is the most appropriate initial management$ %&lease select 1 option' )hlorpheniramine )orrect)orrect &rednisolone -anitidine Topical hydrocortisone Topical mepyramine Frticaria is a common condition and !s!ally responds very well to systemic antihistamines which are the correct first line treatment. +ral steroids can be given for severe cases b!t only as a last resort. Topical steroids<topical antihistamines have no effect. --------------------------------------"ork mart ession - C-)& &art 1 >!estion: 173 of 193 Time taken: 03:1? 5 49-year-old man with a thirty year history of psoriasis presented with generalised erythroderma of three days d!ration. 190

.2amination reveals him to be shivering b!t otherwise well. (e was treated as an inpatient with emollients and attention to fl!id replacement and temperat!re control b!t failed to improve after five days. "hat is the most appropriate ne2t treatment$ %&lease select 1 option' +ral hydro2ychloro#!ine *ncorrect*ncorrect answer selected +ral methotre2ate This is the correct anwserThis is the correct answer +ral prednisolone Topical coal tar Topical ,ithranol .rythroderma is an emergency as patients are s!sceptible to profo!nd dehydration/ infection and hypothermia. Cethotre2ate wo!ld be the only correct treatment for someone with erythrodermic psoriasis. teroids co!ld lead to !nstable p!st!lar psoriasis and wo!ld not generally work. (ydro2ychloro#!ine has little effect on psoriasis. Topical coal tar and ,ithranol are good treatments for chronic pla#!e psoriasis b!t are highly irritant and wo!ld make the erythroderma m!ch more inflamed and deteriorate his condition. ------------------------------>!estion: 179 of 193 Time taken: 03:15

191

5 90-year-old female presents with a si2 month history of pr!ritic pap!les/ vesicles and e2coriations on the elbows/ knees/ b!ttocks and scalp. (er 1& has prescribed topical betamethasone therapy which has been !nhelpf!l. "hat is the most likely diagnosis$ %&lease select 1 option' 5topic dermatitis %eczema' *ncorrect*ncorrect answer selected ,ermatitis herpetiformis %,(' This is the correct anwserThis is the correct answer (enoch- chQnlein p!rp!ra %( &' &soriasis cabies The #!estion describes the characteristic distrib!tion of the lesions of dermatitis herpetiformis. ,( is one of the imm!nob!llo!s conditions and characteristically has very intensely pr!ritic vesicles. *t is not !s!ally responsive to topical steroids/ b!t wo!ld respond well to dapsone. *t is associated with gl!ten sensitivity and coeliac disease. 5topic eczema is non-vesic!lar and wo!ld respond to potent topical steroids. ( & is a p!rp!ric rash and is non-pr!ritic. cabies !s!ally affect the e2tremities and rarely affect above the neck line. They do not ca!se pap!les and vesicles. ----------------------------------->!estion: 175 of 193 Time taken: 03:3? 197

5 45-year-old female presents with chronic leg !lceration which is a conse#!ence of veno!s ins!fficiency. "hich one of the following is the most appropriate management$ %&lease select 1 option' 5ppropriate systemic antibiotic in preparation for skin grafting *ncorrect*ncorrect answer selected )ompression bandaging This is the correct anwserThis is the correct answer *mprove the veno!s ret!rn by limb elevation kin biopsy to e2cl!de neoplasm 0ein s!rgery e2cl!sion of neoplasm by skin biopsy 0eno!s !lcers are secondary to veno!s stasis and chronic stretching vessel of the vein walls of the s!perficial veins. These event!ally become thinner and !lcerate. The only treatment shown in st!dies to be beneficial for this condition wo!ld be to compress the s!perficial veno!s !sing a fo!r layer compression bandage. The patient sho!ld always have their ,oppler@s and 5B&* %ankle brachial p!lse inde2' prior to compression. This sho!ld be greater than 1. ---------------------------------->!estion: 17? of 193 Time taken: 03:54 5 30-year-old woman presents with a skin rash. +n applying press!re to an !naffected area of skin it was relatively easy to ind!ce tra!ma.

193

*ncreased fragility of the skin is characteristic of which of the following conditions$ %&lease select 1 option' 5c!te intermittent porphyria *ncorrect*ncorrect answer selected .pidermolysis b!llosa This is the correct anwserThis is the correct answer =e!rofibromatosis &se!do-2anthoma elastic!m T!bero!s sclerosis *ncreased skin fragility is seen in a n!mber of disorders and is !sed as a clinical test in b!llo!s disorders %=ikolsky@s sign'. +ther ca!ses incl!de: M &emphig!s v!lgaris M &orphyria c!tanea tarda M ,r!g reactions %especially pse!doporphyria'. +ther ca!ses of increased skin fragility %not associated with b!llae' incl!de long term corticosteroid therapy/ .hlers-,anlos syndrome and sc!rvy %vitamin ) deficiency'. --------------------------------->!estion: 174 of 193 Time taken: 09:14 "hich of the following is a feat!re of hereditary haemorrhagic telangiectasia$ %&lease select 1 option' 5 good response to oestrogen therapy *ncorrect*ncorrect answer selected 199

)erebral arterioveno!s malformations This is the correct anwserThis is the correct answer 1* haemorrhage is the !s!al presenting feat!re Telangiectasia of the m!co!s membranes/ b!t not the skin Tendency of lesions to become less obvio!s with age (ereditary haemorrhagic telangiectasia %((T' is a m!ltisystem vasc!lar dysplasia characterised by the presence of m!ltiple arterioveno!s malformations %50Cs' that lack intervening capillaries and res!lt in direct connections between arteries and veins. mall 50Cs/ called telangiectases/ close to the s!rface of skin and m!co!s membranes often r!pt!re and bleed. *t is inherited as an a!tosomal dominant trait. The most common clinical manifestations of ((T are spontaneo!s and rec!rrent epista2is and m!ltiple telangiectases which commonly appear on the lips/ face/ tong!e or hands in ad!lthood. 5 minority of individ!als with ((T have symptomatic gastrointestinal bleeding/ which most commonly begins after age 50 years. :arge 50Cs often ca!se symptoms when they occ!r in the brain or l!ngH complications from bleeding or sh!nting may be s!dden and catastrophic. *t is estimated at least 30I of ((T patients have p!lmonary involvement/ 30I hepatic involvement and 10-70I cerebral involvement. The manifestations of ((T generally develop with age/ and are !s!ally not present at birth. .pista2is is !s!ally the earliest sign of disease/ often occ!rring in childhood. &!lmonary 50Cs can become apparent from p!berty. By the age of 1? years 41I of patients will have developed signs of ((T/ rising to over 80I by 90 years. The )!racao criteria can be !sed to aid diagnosis: The diagnosis is:

195

M ,efinite if three criteria are present M @&ossible@ or @s!spected@ if two criteria are present M @Fnlikely@ if fewer than two criteria are present. )riteria are: M .pista2is: spontaneo!s/ rec!rrent nose bleeds M Telangiectases: m!ltiple/ characteristic sites %lips/ oral cavity/ fingers/ nose' M 0isceral lesions/ s!ch as gastrointestinal telangiectasia/ p!lmonary 50C/ hepatic 50C/ cerebral 50C/ spinal 50C M 6amily history of a first degree relative with ((T. )erebral involvement can be in the form of telangiectasias/ cerebral 50Cs/ ane!rysms or caverno!s angiomas. )erebral 50Cs are tho!ght to affect 10I of patients/ and can res!lt in headaches/ seiz!res/ s!rro!nding ischaemia %steal' or haemorrhage. These lead to significant mortality and morbidity. Canagement of epista2is and gastrointestinal haemorrhage sho!ld be symptomatic initially. "hilst it is generally regarded that p!lmonary 50Cs sho!ld be screened for %and treated with prophylactic antibiotics and embolisation'/ screening for cerebral and hepatic 50Cs remains controversial and is not c!rrently offered in the FD. +estrogen therapy is sometimes advocated b!t its efficacy is !nclear. *t may be beneficial in heavily transf!sion dependent patients. -eference O 6!rther -eading: (ereditary haemorrhagic telangiectasia %+sler-"eber--end! syndrome': a view from the 71st cent!ry. Begbie C./ "allace 1C6 and hovlin ):. &ostgrad Ced G 7003H48:13-79 ------------------------------------->!estion: 173 of 193 Time taken: 09:95 19?

"hich is tr!e regarding eczema herpetic!m$ %&lease select 1 option' *s invariably fatal if !ntreated *ncorrect*ncorrect answer selected *s more severe in reactivation disease *s typically associated with a high fever for over a week This is the correct anwserThis is the correct answer +nly a single crop of vesicles !s!ally appear Fs!ally has an indolent onset .czema herpetic!m is the res!lt of primary infection of eczemato!s skin with (erpes simple2 vir!s %( 0'. The severity varies from mild to fatal. There is !s!ally an abr!pt onset with crops appearing over seven to nine days. These may become coalesced. Typically/ the child has a high fever for seven days/ and rec!rrent attacks can occ!r. ,eath can res!lt from physiological dist!rbances %loss of fl!id electrolytes and protein thro!gh the skin' or dissemination of the vir!s to brain and other organs or from secondary bacterial sepsis. T 7007 ,r )olin Celville ------------------------------>!estion: 178 of 193 Time taken: 09:91 5 1?-year-old boy presents with erythema nodos!m. "hich of the following sho!ld be considered$ %&lease select 1 option' )ytomegalovir!s infection *ncorrect*ncorrect answer selected 194

Dawasaki disease -eiter@s disease To2oplasmosis Flcerative colitis This is the correct anwserThis is the correct answer .rythema nodos!m is characterised by painf!l/ ind!rated/ shiny/ red/ hot/ elevated nod!les 1-3 cm diameter partic!larly on the shins. There may be associated fever/ malaise/ and arthralgia U hilar adenopathy. +ver a period of days they become violaceo!s/ then d!ll p!rple/ then fade like a large br!ise witho!t resid!al !lceration or scar. There may be crops over three to si2 weeks. They are !ncommon !nder the age of ?/ and are commoner in females than males. )a!ses incl!de: 1. *nfections M Bacteria: treptococci/ leptospirosis/ cat-scratch disease/ psittacosis/ Kersinia. M 0ir!ses: .B0. 7. +ther M TB/ t!laraemia/ histoplasmosis/ coccidioidomycosis. 3. ,r!gs M !lphonamides/ oral contraceptive pill. 9. ystemic diseases M :./ vasc!litis/ regional enteritis/ !lcerative colitis/ BehVet syndrome/ sarcoidosis. T 7007 ,r )olin Celville. ---------------------------------------->!estion: 130 of 193

193

Time taken: 09:50 "hich statement regarding tinea capitis is correct$ %&lease select 1 option' *t ca!ses patches that fl!oresce d!ll green !nder "ood@s lamp *ncorrect*ncorrect answer selected *t is effectively treated with topical nystatin ointment *t is most commonly ca!sed by the f!ng!s Trichophyton tons!rans This is the correct anwserThis is the correct answer *t often res!lts in permanent alopecia *ts presence sho!ld s!ggest imm!nological deficiency Tinea capitis is a dermatophyte infection of the scalp. There are a n!mber of ca!sative organsisms/ b!t c!rrently in the FD and F 5 is most often ca!sed by Trichophyton tons!rans/ and occasionally by Cicrospor!m canis. *t is commonest in areas of socio-economic deprivation. There is initially a small pap!le at the base of the hair follicle which spreads peripherally forming a scaly circ!lar pla#!e %ringworm' within which there are brittle/ broken infected hairs %e2clamation mark hairs'. )onfl!ent patches of alopecia develop and there may be pr!ritis. ometimes a severe inflammatory response prod!ces an elevated boggy gran!lomato!s mass %kerion'/ st!dded with sterile p!st!les. There may be fever and regional lymphadenopathy/ and occasionally permanent scarring and alopecia may res!lt. The cr!sted patches fl!oresce d!ll green !nder "ood@s light if ca!sed by Cicrospor!m canis/ b!t do not fl!oresce if ca!sed by Trichophyton tons!rans. Cicroscopic e2amination of a potassi!m hydro2ide %D+(' preparation shows tiny spores and the f!ngi may be grown in abo!ra!d medi!m with antibiotics.

198

+ral griseof!lvin for two to three months is re#!ired/ or ketoconazole for resistant cases. T 7007 ,r )olin Celville. --------------------------------------->!estion: 131 of 193 Time taken: 05:01 "hich of the following s!ggests a diagnosis of moll!sc!m contagios!m rather than chickenpo2$ %&lease select 1 option' 5bsence of erythema s!rro!nding lesions )orrect)orrect :esions disappearing within a month &ositive contact history &resence of mac!les and pap!les &resence of pr!ritis Coll!sc!m contagios!m is ca!sed by a deo2yribon!cleic acid %,=5' po2 vir!s. The lesions are small/ skin colo!red pap!les with central !mbilication. There is little s!rro!nding inflammation and they may be spread following scratching to other sites. )hickenpo2 lesions in the early stages may be mistaken for moll!sc!m. (owever/ the presence of associated mac!les and later vesicles and p!st!les help to differentiate them. These lesions also affect the m!c!s membranes/ and !s!ally disappear within a few weeks/ while moll!sc!m can persist for !p to a year. T 7007 ,r )olin Celville. 150

---------------------------------------------------------->!estion: 137 of 193 Time taken: 05:03 5 7?-year-old man is noted to have cyanosis of the lower limbs and cl!bbing of the toes b!t not the fingers. "hich of the following statements is tr!e$ %&lease select 1 option' (e has coarctation of the aorta *ncorrect*ncorrect answer selected (e has .isenmenger@s syndrome This is the correct anwserThis is the correct answer (e has had a Blalock sh!nt operation (e is likely to have a lo!d contin!o!s @machinery@ m!rm!r below the left clavicle (e is likely to need !rgent s!rgery This is the differential cyanosis of a reversed patent d!ct!s arterios!s %&,5'. There is a right-left sh!nt from the p!lmonary artery to the aorta A!st distal to the left s!bclavian artery. )oarctation ca!ses radiofemoral delay. *t may be associated with &,5 b!t there is no s!ggestion in this patient. )ontin!o!s machinery m!rm!r is the classic m!rm!r of &,5 b!t when the sh!nt reverses %as in patients with a large &,5 and<or p!lmonary disease' the m!rm!r becomes softer and shorter. "hen .isenmenger@s syndrome has developed s!rgery is associated with a very high mortality.

151

5 Blalock sh!nt %anastomosis of s!bclavian artery to p!lmonary artery' !sed to be performed for 6allot@s tetralogy and leads to a weak left radial p!lse. ------------------------------------->!estion: 133 of 193 Time taken: 05:05 "hich of the following is tr!e of c!taneo!s anthra2$ %&lease select 1 option' )a!ses a black eschar which overlies p!s *ncorrect*ncorrect answer selected *s very likely to occ!r in s!bAects e2posed to anthra2 spores :esions are associated with marked oedema This is the correct anwserThis is the correct answer :esions are !s!ally painf!l and tender Cortality is appro2imately 70I despite antibiotic therapy 5nthra2 is ca!sed by Bacill!s anthracis a 1ram positive rod. )!taneo!s anthra2 is ca!sed by direct contact of the bacteria into an open wo!nd %!s!ally to!ching an infected animal'. )!taneo!s anthra2 is associated with a black eschar witho!t p!s/ tends to be painless and to have widespread oedema. "itho!t antibiotics mortality is of the order of 70I/ b!t with antibiotics/ mortality is low/ which contrasts with p!lmonary anthra2. 6!rther reading: 5merican 5cademy of ,ermatology. )!taneo!s 5nthra2 Canagement 5lgorithm %accessed 19 =ovember 7017' -------------------------------------->!estion: 139 of 193 157

Time taken: 0?:0? "hich of the following concerning leg !lcers is correct$ %&lease select 1 option' ,i!retics have been shown to improve !lcer healing when associated with oedema *ncorrect*ncorrect answer selected *n diabetic !lcers/ the dressing sho!ld be left in sit! for no more than one week This is the correct anwserThis is the correct answer :arge gravitational !lcers are always painf!l Treating s!perficial infection with antibiotics has been shown to be beneficial Flcers ca!sed by arterial disease are typically treated by compression bandaging ,i!retics may red!ce oedema b!t have not been demonstrated per se to red!ce healing time. 1ravitational !lcers are not !s!ally painf!l. *f there are no obvio!s feat!res of s!rro!nding cell!litis/ antibiotic therapy is !s!ally !nnecessary and has not been shown to improve healing in s!perficial infection which is common in !lceration. ---------------------------------------------->!estion: 135 of 193 Time taken: 0?:05 "hich of the following is commonly associated with psoriasis$ %&lease select 1 option' 5ng!lar stomatitis *ncorrect*ncorrect answer selected

153

Doebner phenomenon This is the correct anwserThis is the correct answer +ptic ne!ritis -esponse to chloro#!ine carring alopecia &soriasis is a chronic relapsing inflammatory skin disorder most commonly characterised by erythemato!s/ sharply demarcated pap!les and ro!nded pla#!es covered by silvery scales. ,iagnosis is !s!ally clinical/ and skin biopsy is rarely re#!ired to confirm psoriasis. =ew lesions often appear at sites of inA!ry or tra!ma %Doebner phenomenon'/ which typically occ!rs one to two weeks after the skin has been damaged. &soriasis can be associated with an anterior !veitis/ b!t optic ne!ritis is not a recognised complication. 5ng!lar stomatitis describes erythema and fiss!ring of the skin adAacent to the angle of the mo!th. The most common ca!se is )andida infection/ b!t it is also associated with allergy/ seborrhoeic dermatitis/ vitamin B deficiencies and iron deficiency. *t is not commonly described in association with psoriasis. The scalp is often involved in psoriasis/ especially in children and adolescents. Cost commonly it ca!ses a telogen effl!vi!m/ that is/ the hair follicles are forced into the telogen resting stage. *t is rare for psoriasis to ca!se a scarring alopecia. Fp to 30I of patients with chronic pla#!e psoriasis may be affected by an arthropathy. This can range from mild distal interphalangeal Aoint involvement with nail pitting to severe arthritis m!tilans.

159

"hilst the e2act ca!se is !nknown/ psoriasis has a strong genetic basis. .!ropean pop!lations are commonly affected/ and there are two peaks of incidence at 1?-77 years and 54-?0 years. Cales and females are e#!ally affected. .2ternal factors s!ch as infection/ stress and medication may e2acerbate psoriasis. ome of the common medications associated with triggering or worsening psoriasis incl!de lithi!m/ gold salts/ beta-blockers and antimalarials %incl!ding chloro#!ine'. -eferences: ,r!g ind!ced psoriasis. Cilavec-&!retic 0. et al. 5cta ,ermatovenerol )roat. 7011 CarchH18%1':38-97 --------------------------------------------->!estion: 13? of 193 Time taken: 0?:78 "hich of the following concerning pityriasis rosea is correct$ %&lease select 1 option' *t is characterised by flat scaly patches )orrect)orrect *t is d!e to a f!ngal infection *t is fre#!ently associated with oro-genital itching Cay be preceded by intense itching Tends to rec!r after apparent c!re &ityriasis rosea is a rash that can occ!r at any age/ b!t it occ!rs most commonly in people between the ages of 10 and 35 years. *t may be set off by a viral infection b!t does not appear to be contagio!sH herpes vir!ses ? and 4 have most often been associated with pityriasis rosea. *t is not ca!sed by a f!ng!s. 155

*t is not related to foods/ medicines/ or stress. *t most often affects teenagers or yo!ng ad!lts. The condition often begins as a large single pink patch on the chest or back. This patch may be scaly and is called a @herald@ or @mother@ patch. "ithin a week or two/ more pink patches/ sometimes h!ndreds of them/ appear on the body and on the arms and legs. &atches may also occ!r on the neck/ and tho!gh rare/ the face. The oval patches follow the line of the ribs like a fir tree. They have a dry s!rface and may have an inner circlet of scaling. -------------------------------------->!estion: 134 of 193 Time taken: 0?:34 "hich of the following may be responsible for an ac!te relapse of systemic l!p!s erythematos!s % :.' in a 33-year-old female$ %&lease select 1 option' (ydralazine therapy *ncorrect*ncorrect answer selected &regnancy This is the correct anwserThis is the correct answer &rogesterone only contraceptive pill almeterol therapy "inter holiday in :apland ome physiological and environmental factors affect the periods of deterioration and of remission in systemic l!p!s erythematos!s. These factors incl!de hormone replacement therapy %(-T' and partic!larly the oral contraceptive/ pregnancy and infection. *t wo!ld not be e2pected with the progesterone only oral contraceptive. 15?

Ko! wo!ld e2pect to find virt!ally no s!n on a winter holiday in :apland %5rctic circle'. 5 n!mber of dr!gs %hydralazine/ procainamide/ isoniazid/ chlorpromazine/ ,penicillamine and methyldopa' can res!lt in dr!g-ind!ced l!p!s in predisposed individ!als. This can be differentiated from the idiopathic :. on genetic and imm!nologic gro!nds. 6!rthermore/ M *t is mild and reversible on stopping the dr!g M -enal disease and do!ble stranded anti-,=5 are rare %altho!gh antibodies specific for histones may be present' M The se2 ratio is e#!al. These dr!gs do not ca!se deterioration in patients with :.. -------------------------------------------->!estion: 133 of 193 Time taken: 0?:33 5 93-year-old woman with atopic dermatitis %atopic eczema' presented with an ac!te generalised e2acerbation of her disease. he was admitted to hospital b!t failed to improve with emollients/ topical betamethasone 14-valerate and oral antihistamine. "hich one of the following dr!gs is the most appropriate treatment$ %&lease select 1 option' 5citretin *ncorrect*ncorrect answer selected 154

5mo2icillin )olchicine )yclosporin This is the correct anwserThis is the correct answer ,apsone )yclosporin is a well !sed dr!g in the treatment of atopic dermatitis %5,'. *t is !s!ally at doses of 7-5 mg<kg. The pathophysiology of 5, is comple2 b!t the T lymphocytes are involved and it is known that there is an increased prod!ction of cytokines partic!larly *:-9. )yclosporin is a s!ppressor of T cells and in that respect works very well in atopic dermatitis and psoriasis. The side effects of hypertension and renal to2icity limit its !se. These patients are seen monthly to have their blood press!re and !rea and electrolytes checked. -------------------------------------->!estion: 138 of 193 Time taken: 0?:78 5 45-year-old female presents with generalised erythema and p!st!le formation. he has a history of psoriasis and has recently been treated with oral prednisolone for asthma. "hat is the most appropriate ne2t co!rse of action$ %&lease select 1 option' 5dmission to hospital )orrect)orrect &atch testing 153

&soralen with !ltraviolet-5 therapy %&F05' kin biopsy Treatment with erythromycin as an o!tpatient This is erythroderma which is a dermatological emergency. The patient needs admission with close s!pervision and s!pportive treatment with *0 fl!ids and antibiotics. ---------------------------------------->!estion: 190 of 193 Time taken: 0?:34 5 77-year-old woman complains of haemoptysis/ abdominal pains and pyre2ia for a month. he is admitted to hospital and fo!nd to be apyre2ial and haemodynamically stable. There are n!mero!s cr!sted/ linear lesions on her forearms. "hat is the most likely diagnosis$ %&lease select 1 option' 5c!te intermittent porphyria *ncorrect*ncorrect answer selected 6actitio!s disorder This is the correct anwserThis is the correct answer ystemic l!p!s erythematos!s TB "egener@s gran!lomatosis This lady warrants f!rther investigation/ b!t the #!estion asks yo! what the most likely diagnosis is. The history is very vag!e and on clinical e2amination she has no clinical feat!res other than a rash. he is apyre2ial on this occasion/ and the fact she is haemodynamically stable r!les o!t prolonged significant haematemesis. :inear lesions are rarely ca!sed by

158

organic disease/ and sho!ld make yo! #!ery dermatitis artifacta especially when the lesions are in accessible locations s!ch as the forearms. &orphyria may be e2pected to have vesicles on s!n e2posed regions/ s!ch as the face/ forearms and legs. (owever/ the history is too long for ac!te intermittent porphyria. 5ttacks typically present with a!tonomic dist!rbance and sympathetic overactivity/ and often with ne!rological signs/ none of which are present here. The history co!ld fit with TB/ if the patient has risk factors for e2pos!re. .arly morning sp!t!m or !rine co!ld be e2amined to help e2cl!de this diagnosis. 5 chest radiograph may be indicated. ystemic l!p!s erythematos!s can present in a variety of ways/ and can occ!r in this age gro!p. 5 more detailed history wo!ld be re#!ired/ and testing 5=5 %anti-n!clear antibodies' may be helpf!l. "egener@s gran!lomatosis typically has a more ac!te presentation than this/ with evidence of hard clinical signs. hortness of breath is !s!ally evident if p!lmonary haemorrhage has developed. in!s symptoms are often also present. ------------------------------------------------->!estion: 191 of 193 Time taken: 04:04 5 95-year-old woman is admitted with a spiking temperat!re and sweats. he has been !nwell for the last three weeks with flitting arthralgia and lethargy. There is a rash over her tr!nk which is most prevalent in the mornings.

1?0

Blood c!lt!res are sterile. (er recent transthoracic echocardiogram is normal. . - is 5? mm<ho!r. (er ferritin is elevated at ?000 mg<l. 5!toimm!ne screen is negative. "hat is the likely diagnosis$ %&lease select 1 option' 5d!lt onset till@s disease )orrect)orrect Bacterial endocarditis Ceningitis -he!matoid arthritis ystemic l!p!s erythematos!s till@s disease is a febrile syndrome in yo!ng ad!lts %1?-35 years' which affects m!ltiple organs.The diagnosis is mainly one of e2cl!sion. The clinical feat!res incl!de: M (igh spiking fever %once a day/ with ret!rn of temperat!re to normal' M 5rthralgia<arthritis M ore throat M Transient mac!lopap!lar rash %mildly pr!ritic in 1<3' M :ymphadenopathy M (epatosplenomegaly and M &le!ritis<pericarditis. -arely there may be M 5septic meningitis M )ranial nerve palsies M *ritis and M &eripheral ne!ropathy. There is often delay in diagnosis. 1?1

(yperferritinaemia %greater than five times normal' is present in 80I of cases. ------------------------------->!estion: 197 of 193 Time taken: 04:73 5 90-year-old man presented with pityriasis versicolor. "hat is the most appropriate treatment$ %&lease select 1 option' Cethotre2ate *ncorrect*ncorrect answer selected +ral terbinafine &hototherapy with !ltraviolet light %F0B' &soralen with !ltraviolet light %&F05' therapy Topical seleni!m s!lphide This is the correct anwserThis is the correct answer &ityriasis versicolor %also called tinea versicolor' is a skin lesion ca!sed by a f!ng!s called Calassezia f!rf!r. The treatment is topical seleni!m s!lphide. +ral itraconazole is also effective. ------------------------------------>!estion: 193 of 193 Time taken: 04:75 "hat is the most common presenting feat!re of porphyria c!tanea tarda$ %&lease select 1 option' 1?7

5c!te blistering crises affecting the tr!nk and limbs *ncorrect*ncorrect answer selected 5c!te redness and swelling following s!n e2pos!re .rythroderma 1eneralised hypertrichosis kin fragility and blistering affecting the hands/ face and scalp This is the correct anwserThis is the correct answer &orphyria c!tanea tarda %&)T' is a term that encompasses a gro!p of related disorders/ all of which arise from deficient activity of the haeme-synthetic enzyme !roporphyrinogen decarbo2ylase %F-+-,' in the liver. The porphyrins prod!ced in &)T are photoactive molec!les that absorb light energy strongly in the visible violet spectr!m. &hotoe2cited porphyrins in the skin mediate o2idative damage to biomolec!lar targets/ ca!sing c!taneo!s photosensitivity reactions. The most common presenting sign of &)T is fragility of s!ne2posed skin after mechanical tra!ma/ leading to erosions and b!llae/ worst on dorsal hands/ forearms and face. ------------------------------------------->!estion: 199 of 193 Time taken: 04:35 ,eficiency of which one of the following trace elements is implicated as a ca!se of cardiomyopathy$ %&lease select 1 option' )hromi!m *ncorrect*ncorrect answer selected )opper Canganese eleni!m This is the correct anwserThis is the correct answer Einc 1?3

eleni!m deficiency is one of the reversible ca!ses of dilated cardiomyopathy. ---------------------------------------->!estion: 195 of 193 Time taken: 03:01 "hich of the following is aggravated by e2pos!re to s!nlight$ %&lease select 1 option' 5cne v!lgaris *ncorrect*ncorrect answer selected 5c!te intermittent porphyria &ellagra This is the correct anwserThis is the correct answer &se!do2anthoma elastic!m &soriasis .2acerbation or localisation of other dermatoses is characteristic of: M &ellagra M (artn!p@s disease M :!p!s erythematos!s M ,arier@s disease M -osacea M cleroderma M 5ctinic lichen plan!s M :ymphocytoma. -------------------------------------------->!estion: 19? of 193 Time taken: 03:11 5 73-year-old female presents with a problem with her nails.

1?9

+ver the last two months they have become rather !nslightly and brittle. he has taken a selection of medications for acne. .2amination reveals onycholysis. "hich of the following preparations is most likely to be responsible for the onycholysis$ %&lease select 1 option' ,ianette *ncorrect*ncorrect answer selected .rythromycin *sotretinoin Tetracycline This is the correct anwserThis is the correct answer Topical benzoic acid Tetracycline is a recognised ca!se of onycholysis together with eczema/ psoriasis and thyroto2icosis to name b!t a few. ------------------------------------------------------->!estion: 194 of 193 Time taken: 03:11 5 73-year-old obese female with known t!berc!losis presents with !lcerating nod!les on the back of her legs. "hich of the following is the most likely diagnosis$ %&lease select 1 option' .rythema ind!rat!m %.*' )orrect)orrect .rythema marginat!m .rythema nodos!m :!p!s pernio :!p!s v!lgaris

1?5

.* is a form of pannic!litis characterised by chronic/ rec!rrent/ tender/ s!bc!taneo!s/ and sometimes !lcerated nod!les on the lower legs that may also appear elsewhere. 6emales are more fre#!ently affected/ with a female:male ratio of 4:1 and it is more fre#!ent in yo!nger females. *t is fo!nd in association with t!berc!losis. .rythema nodos!m also a pannic!litis is also commonly associated with t!berc!losis and presents with painf!l erythemato!s nod!les in rec!rrent crops over the legs and arms. The lesions however do not !lcerate. 5nother possibility altho!gh not provided in the stems is pyoderma gangrenos!m. +n the other hand/ l!p!s v!lgaris is a chronic/ progressive and destr!ctive form of c!taneo!s t!berc!losis in patients with a moderate or high degree of imm!nity. *t occ!rs more commonly in females than in males. The classical lesions consist of reddish-brown pla#!es not nod!les. The lesions progress by peripheral e2tension and central healing/ atrophy and scarring. The areas of predilection are head and neck %30I'/ followed by arms/ legs/ then tr!nk. This rash is not the typical description of erythema marginat!m %finer rash' or m!ltiforme %blisters/ targets'. :!p!s pernio occ!rs in association with sarcoid. --------------------------------------------->!estion: 193 of 193 Time taken: 03:71

1??

5 79-year-old female presents with vag!e frontal headaches and vis!al dist!rbance. he has a past history of acne for which she is receiving treatment. .2amination reveals her to be obese with a blood press!re of 110<40 mm(g. There is absence of the central retinal vein p!lsation on f!ndoscopic e2amination. "hich of the following dr!gs acco!nt for these findings$ %&lease select 1 option' 5mpicillin *ncorrect*ncorrect answer selected ,ianette This is the correct anwserThis is the correct answer .rythromycin *sotretinoin Topical tetracycline ,ianette/ like any oral contraceptive/ may be associated with benign intracranial hypertension %B*('. Topical tetracycline is not associated with B*(. -arely B*( has been associated with isotretinoin b!t !s!ally in combination with a tetracycline.

1?4

Das könnte Ihnen auch gefallen